LinAlg2/LinAlg2.tex

1572 lines
81 KiB
TeX

\documentclass[12pt, a4paper]{report}
\usepackage{amsmath}
\usepackage{amsfonts}
\usepackage{enumitem}
\usepackage{amsthm}
\usepackage{amssymb}
\usepackage[dvipsnames]{xcolor}
\usepackage{marvosym}
\usepackage{mathtools}
\usepackage[colorlinks=true, linkcolor=magenta]{hyperref}
\usepackage{cancel}
\usepackage[ngerman]{babel}
\title{Lineare Algebra 2}
\date{Sommersemester 2022}
\author{Philipp Grohs \\ \small \LaTeX-Satz: Anton Mosich}
\newtheoremstyle{theostyle}%
{3pt}%
{3pt}%
{}%
{}%
{\bfseries}%
{:}%
{\newline}%
{}%
\theoremstyle{theostyle}
\newtheorem{theo}{Theorem}[section]
\newtheorem{lemma}[theo]{Lemma}
\newtheorem{defin}[theo]{Definition}
\newtheorem{satz}[theo]{Satz}
\newtheorem{korollar}[theo]{Korollar}
\newtheorem{folgerung}[theo]{Folgerung}
\DeclareMathOperator{\sgn}{sgn}
\DeclareMathOperator{\spec}{spec}
\DeclareMathOperator{\spur}{sp}
\DeclareMathOperator{\homk}{Hom_\mathbb{K}(V, V)}
\DeclareMathOperator{\homkk}{Hom_\mathbb{K}}
\DeclareMathOperator{\adj}{adj}
\DeclareMathOperator{\id}{id}
\DeclareMathOperator{\diag}{diag}
\DeclareMathOperator{\K}{\mathbb{K}}
\DeclareMathOperator{\eig}{Eig}
\DeclareMathOperator{\nxn}{n \times n}
\DeclareMathOperator{\im}{im}
\begin{document}
\maketitle
\tableofcontents
\chapter{Determinanten}
\section{Permutationen}
\begin{defin}
Sei $n \in \mathbb{N} \setminus \{0\}, [n] := \{1, 2, \dots, n\}$. \\
Eine bijektive Abbildung $\pi:[n]\to[n]$ heißt \underline{Permutation} von $[n]$.
Wir definieren die \underline{symmetrische Gruppe}
$S_n := \{\pi\text{ Permutation von }[n]\}$
mit der Hintereinanderausführung als Gruppenoperation.
\end{defin}
\subsubsection{Bemerkung}
\begin{itemize}
\item $(S_n, \circ)$ ist eine Gruppe.
\item $\pi\in S_n$ ist eindeutig durch das Tupel $(\pi(1), \dots, \pi(n))$ definiert.
\item Fixpunkte $(\pi(i)=i)$ werden oft weggelassen.
\end{itemize}
\begin{defin}
$\pi\in S_n$ heißt \underline{Transposition} wenn es $i, j\in [n]$ gibt mit
\[\pi(k) = \begin{cases} k & k\notin\{i, j\}\\ i & k = j\\ j & k=i \end{cases}\]
Wir schreiben $\pi = (ij)$.
\end{defin}
\begin{satz} \label{theo:1.1.3}
Es gilt $\lvert S_n \rvert = n!$.
\begin{proof}
Vollständige Induktion
\begin{itemize}
\item $n=1: S_1 = \{\id\}\implies\lvert S_1\rvert = 1 = 1!$
\item $n-1\to n:$\\ Angenommen $\lvert S_{n-1} \rvert = (n-1)!$. Dann gilt $\lvert\{\pi \in S_n: \pi(n) = n\}\rvert = (n-1)!$. Sei allgemein $i\in[n]$. Dann gilt $\pi(n)=i \iff (in)\circ\pi(n)=n$. Also gilt
\begin{align*}
& \lvert\{\pi\in S_n: \pi(n)=i\}\rvert = \lvert\{(in)\circ\pi: \pi(n)=n\}\rvert \\
& = \lvert\{\pi: \pi(n)=n\}\rvert = (n-1)!
\end{align*}
Weiters gilt
\begin{align*}
& S_n = \bigcup_{i\in[n]}^\bullet\{\pi\in S_n: \pi(n)=i\} \implies \\
& \lvert S_n\rvert = \sum_{i\in[n]}\lvert\{\pi \in S_n: \pi(n) = i\}\rvert
= n\cdot(n-1)! = n!
\end{align*}
\end{itemize}
\end{proof}
\end{satz}
\begin{satz} \label{theo:1.1.4}
Für $n\in \mathbb{N}_{\ge2}$ ist jedes $\pi \in S_n$ das Produkt von (endlich vielen) Transpositionen.
\begin{proof}
\begin{itemize}
\item $n=2: S_2 = \{\id, (2 1)\}$
\item $n-1\to n$\\
Sei $\pi \in S_n$. Dann gilt (siehe Beweis von Satz \ref{theo:1.1.3}) mit $i=\pi(n)$, dass
\[\underbrace{(i n)\pi}_{\pi_i}(n) = n\]
Sei $\pi_i = (\underbrace{\pi_i(1) \dots \pi_i(n-1)}_{\in S_{n-1}} n) \underset{\text{Induktions VS}}{\implies} \pi_i = (i_1 j_1) \dots (i_k j_k)$.\\
Außerdem gilt $\pi = (i n)\pi_i$, also $\pi = (i n)(i_1 j_1) \dots (i_k j_k)$
\end{itemize}
\end{proof}
\end{satz}
\subsubsection{Bemerkung}
\begin{itemize}
\item Produktdarstellung ist nicht eindeutig, zum Beispiel:\\ $(3 1 2) = (2 1)(3 1) = (3 1)(3 2)$
\item $f\in \mathbb{Z}[X_1, \dots, X_n], \pi \in S_n$\\$\pi f(X_1, \dots, X_n) := f(X_{\pi(1)}, \dots, X_{\pi(n)})$
\end{itemize}
\subsubsection{Beispiel}
$\pi = (2 3 1), f(X_1, X_2, X_3) = X_1-X_2+X_1X_3 \implies \pi f(X_1, X_2, X_3) = X_2 - X_3 + X_2X_1$
\begin{lemma} \label{theo:1.1.5}
Sei $f(X_1, \dots, X_n) = \prod\limits_{\substack{i, j\in[n]\\ i < j}} (X_j-X_i)\in \mathbb{Z}[X_1, \dots, X_n]$.\\
Dann gilt \begin{enumerate}[label=\alph*)]
\item Zu jedem $\pi \in S_n$ existiert eine eindeutig Zahl $s(\pi) \in \{-1, 1\}$ mit $\pi f = s(\pi)f$.
\item Für $\pi$ eine Transposition gilt $s(\pi) = -1$.
\end{enumerate}
\begin{proof}
\begin{enumerate}[label=\alph*)]
\item \begin{equation*}\begin{aligned}
\pi f(X_1, \dots, X_n) & = \prod_{i<j}(X_{\pi(j)}-X_{\pi(i)}) \\
& =\Bigg(\prod_{\substack{i<j \\\pi(i)<\pi(j)}}(X_{\pi(j)}-X_{\pi(i)})\Bigg)\Bigg(\prod_{\substack{i<j\\\pi(j)<\pi(i)}}(X_{\pi(i)}-X_{\pi(j)})\Bigg) \\
& = (-1)^{\lvert\{(i, j)\in[n]\times[n]:i<j\land\pi(i)>\pi(j)\}\rvert}\prod_{i<j}(X_j-X_i) \\
& = s(\pi)f(X_1, \dots, X_n) \text{ mit } \\
s(\pi) & = (-1)^{\lvert\{(i, j)\in[n]\times[n]:i<j\and\pi(i)>\pi(j)\}\rvert}
\end{aligned}\end{equation*}
\item $\pi = (i j), i<j, k\in\{i+1, \dots, j-1\}$:
$\pi(i, j) = (j, i), \pi(i, k) = (j, k), \pi(k, j) = (k, i)$\\
Für diese Paare gilt $x<y \land \pi(x) > \pi(y)$\\
Für alle anderen Paare gilt $x<y \land \pi(x)<\pi(y)$\\
Erstere sind $2(j-i-1)+1$ Paare. Daraus folgt $\pi f=(-1)^{2(j-i-1)+1}f$, also $s(\pi)=-1$.
\end{enumerate}
\end{proof}
\end{lemma}
\begin{defin}
\leavevmode
\begin{itemize}
\item Die durch Lemma \ref{theo:1.1.5} bestimmte Größe $s(\pi)$ heißt \underline{Signum} von $\pi \in S_n$. Wir schreiben $\sgn(\pi)$.
\item $\pi$ heißt \underline{gerade} falls $\sgn(\pi)=1$ und \underline{ungerade} falls $\sgn(\pi)=-1$.
\end{itemize}
\end{defin}
\begin{satz} \label{theo:1.1.7}
Für $\pi, \sigma \in S_n$ gilt \[\sgn(\sigma\pi)=\sgn(\sigma)\sgn(\pi)\]
\begin{proof}
Nach Satz \ref{theo:1.1.5}(a) gilt:
\begin{align*}
& f(X_1, \dots, X_n) = \prod\limits_{i<j}(X_j-X_i) \implies \\
& \sigma\pi f(X_1, \dots, X_n) = \sgn(\sigma\pi)f(X_1, \dots, X_n)
\end{align*}
Andererseits gilt: \begin{equation*}\begin{split}\sigma\pi f(X_1, \dots, X_n) &= \sigma[\pi f(X_1, \dots, X_n)]\\
&= \sigma[\sgn(\pi)f(X_1, \dots, X_n)] \\
&= \sgn(\pi) \sigma f(X_1, \dots, X_n) \\
&= \sgn(\pi)\sgn(\sigma)f(X_1, \dots, X_n)
\end{split}
\end{equation*}
\end{proof}
\end{satz}
\begin{satz}
\leavevmode
\begin{enumerate}[label=\alph*)]
\item $\sgn(\pi)=1\iff\pi$ ist Produkt gerader Anzahl Transpositionen
\item $\pi$ Produkt von k Transpositionen $\implies \sgn(\pi)=(-1)^k$
\end{enumerate}
\begin{proof}
Folgt direkt aus Satz \ref{theo:1.1.5}(b) und Satz \ref{theo:1.1.7}
\end{proof}
\end{satz}
\begin{folgerung}
Es gibt genau $\frac12n!$ gerade und $\frac12n!$ ungerade Permutationen in $S_n$
\begin{proof}
Folgt aus Satz \ref{theo:1.1.3}
\end{proof}
\end{folgerung}
\begin{defin}
Die geraden Permutationen bilden eine Untergruppe $A_n$ von $S_n$, die man \underline{alternierende Gruppe} nennt.
\end{defin}
\section{Multilinearformen}
\begin{defin}
Seien $V_1, \dots, V_n, W \mathbb{K}-$Vektorräume. Eine Abbildung $\varphi: V_1 \times \dots \times V_n \to W$ heißt \underline{n-linear}, wenn für alle $v_1, v'_1 \in V_1, \dots, v_n, v'_n\in V_n, i \in [n], \lambda\in\mathbb{K}$ gilt, dass
\begin{itemize}
\item $\varphi(v_1, \dots, v_i+v'_i, \dots, v_n)=\varphi(v_1, \dots, v_i, \dots, v_n)+\varphi(v_1, \dots, v'_i, \dots, v_n)$
\item $\varphi(v_1, \dots, \lambda v_i, \dots, v_n)= \lambda\varphi(v_1, \dots, v_i, \dots, v_n)$.
\end{itemize}
Ist $W=\mathbb{K}$ und $V_1, \dots, V_n=V$, so heißt $\varphi$ \underline{n-Linearform}. ($n=2 \to$ \underline{Bilinearform})
\end{defin}
\subsubsection{Beispiel}
\[
\varphi:
\begin{cases}
\mathbb{K}^2\times \mathbb{K}^2 & \to \mathbb{K} \\
\left(\begin{pmatrix}a_{11}\\a_{21}\end{pmatrix}, \begin{pmatrix}a_{12}\\a_{22}\end{pmatrix}\right)
& \mapsto a_{11}a_{22} - a_{12}a_{21}
\end{cases}
\]
\begin{defin} \label{theo:1.2.2}
Eine n-Linearform von V heißt
\begin{itemize}
\item \underline{nicht ausgeartet}, falls
$(a_1, \dots, a_n)\in V\times\dots\times V$ existiert mit \\
$\varphi(a_1, \dots, a_n) \neq 0$.
\item \underline{alternierend}, falls $\varphi(a_1, \dots, a_n)=0$ für $a_1, \dots, a_n$ linear abhängig.
\end{itemize}
\end{defin}
\subsubsection{Bemerkung}
$\varphi$ alternierend und $a_i = a_j$ für $i\neq j \implies \varphi(a_1, \dots, a_n) = 0$.
\begin{lemma} \label{theo:1.2.3}
Sei $\varphi$ alternierende n-Linearform von V und $\pi \in S_n$. Dann gilt für\\
$a_1, \dots, a_n\in V$:
\[\varphi(a_{\pi(1)}, \dots, a_{\pi(n)})=\sgn(\pi)\varphi(a_1, \dots, a_n)\]
\begin{proof}
Wegen Satz \ref{theo:1.1.4} und Satz \ref{theo:1.1.7} genügt es anzunehmen, dass $\pi$ Transposition ist. Sei also $\pi=(ij)$. Es gilt
\begin{equation*}
\begin{aligned}
0 & =\varphi(a_1, \dots, \underbrace{a_i+a_j}_{i}, \dots, \underbrace{a_i+a_j}_{j}, \dots, a_n) \\
& =\underbrace{\varphi(a_1, \dots, a_i, \dots, a_i, \dots, a_n)}_{0} + \underbrace{\varphi(a_1, \dots, a_j, \dots, a_j, \dots, a_n)}_{0} \\ &+ \varphi(a_1, \dots, a_i, \dots, a_j, \dots, a_n) + \varphi(a_1, \dots, a_j, \dots, a_i, \dots, a_n) \\
& \implies \varphi(a_1, \dots, a_j, \dots, a_i, \dots, a_n)=\underbrace{(-1)}_{=\sgn{\pi}}\varphi(a_1, \dots, a_i, \dots, a_j, \dots, a_n)
\end{aligned}
\end{equation*}
\end{proof}
\end{lemma}
\begin{lemma} \label{theo:1.2.4}
Sei $V$ $\mathbb{K}$-VR mit $\dim(V)=n$ und $\varphi$ nicht ausgeartete und alternierende n-Linearform von V. Dann gilt
\[a_1, \dots, a_n \text{ linear abhängig} \iff \varphi(a_1, \dots, a_n) = 0\]
\begin{proof}
\begin{itemize}
\item[$\implies$]: folgt aus Definition \ref{theo:1.2.2}
\item[$\impliedby$]: z.Z.: $\varphi(b_1, \dots, b_n)\neq0\impliedby b_1, \dots, b_n \text{ Basis von } V$. Da $\varphi$ nicht ausgeartet ist, gibt es $a_1, \dots, a_n\in V$ mit $\varphi(a_1, \dots, a_n)\neq0$.\\
Da $b_1, \dots, b_n$ Basis gibt es $\lambda_{ij}\in\mathbb{K}$ mit $a_i=\sum\limits_{j=1}^n{\lambda_{ij}b_j}$\\
Wegen n-Linearität gilt
\[
\begin{aligned}
0\neq\varphi(a_1, \dots, a_n) & =\sum_{j_1=1}^n{\dots}\sum_{j_n=1}^n{\varphi(b_{j_1}, \dots, b_{j_n})\lambda_{1j_1}\cdots\lambda_{nj_n}} \\
& \underbrace{=}_{\varphi\text{ alternierend}}
\sum_{\substack{j_1, \dots, j_n \\\text{paarweise verschieden}}}
{\varphi(b_{j_1}, \dots, v_{j_n})\lambda_{1j_1} \cdots \lambda_{nj_n}} \\
& = \sum_{\pi\in S_n} \varphi(b_{\pi(1)}, \dots, b_{\pi(n)})\lambda_{1\pi(1)} \cdots \lambda_{n\pi(n)} \\
& \underbrace{=}_{\text{Lemma \ref{theo:1.2.3}}}\varphi(b_1, \dots, b_n)\Big(\sum_{\pi\in S_n}\sgn(\pi)\lambda_{1\pi(1)}\cdots\lambda_{n\pi(n)}\Big) \\
& \implies\varphi(b_1, \dots, b_n)\neq0
\end{aligned}
\]
\end{itemize}
\end{proof}
\end{lemma}
\begin{satz} \label{theo:1.2.5}
Sei V $\mathbb{K}$-VR mit $\dim(V)=n$ und Basis $a_1, \dots, a_n$.
\begin{enumerate}[label=\alph*)]
\item Für $\varphi$ alternierende nicht ausgeartete n-Linearform gilt für\\
$b_i = \sum\lambda_{ij}a_j$, dass
\[\varphi(b_1, \dots, b_n) = \varphi(a_1, \dots, a_n)(\sum_{\pi \in S_n}\sgn(\pi)\lambda_{1\pi(1)}\cdots\lambda_{n\pi(n)})
\]
\item Sei $c\in\mathbb{K}\setminus\{0\}$. Dann ist die Abbildung
\[
\varphi(b_1, \dots, b_n) = c(\sum_{\pi \in S_n}\sgn(\pi)\lambda_{1\pi(1)}\cdots\lambda_{n\pi(n)})
\]
eine alternierende nicht ausgeartete n-Linearform.
\end{enumerate}
\begin{proof}
\begin{enumerate}[label=\alph*)]
\item folgt aus dem Beweis von Lemma \ref{theo:1.2.4}.
\item Man verifiziert leicht, dass $\varphi$ n-linear ist. Weiters ist $\varphi$ nicht ausgeartet, da
\[
\varphi(a_1, \ldots, a_n) = c(\sum_{\pi\in S_n}\sgn(\pi)\delta_{1\pi(1)}, \cdots, \delta_{n\pi(n)})=c\cdot1\neq0
\]
z.Z.: $\varphi$ alternierend. Seien $b_1, \dots, b_n$ linear abhängig.\\
O.B.d.A. $b_1=\mu_2b_2+\cdots+\mu_nb_n$. Dann gilt
\[\varphi(b_1, \dots, b_n) = \sum_{j=2}^{n}\mu j \varphi(b_j, b_2, \dots, b_n)\]
Es genügt also zu zeigen, dass $\varphi(b_1, \dots, b_n) = 0$ falls $b_1 = b_i, i\in\{2, \dots, n\}$.
Dann gilt aber $\lambda_{1j}=\lambda_{ij} \forall j$.
\[
\begin{aligned}
\varphi(b_i, \dots, b_i, \dots, b_n) & = c\cdot\sum_{\pi\in S_n} \sgn(\pi) \lambda_{i\pi(1)}\cdots\lambda_{i\pi(i)}\cdots\lambda_{n\pi(n)} \\
& =c\cdot \Bigg(\sum_{\pi\in A_n}\sgn(\pi)\lambda_{i\pi(i)}\cdots\lambda_{i\pi(i)}\cdots\lambda_{n\pi(n)} \\
& +\sum_{\pi\in A_n}\underbrace{\sgn(\pi\circ(1i))}_{=-\sgn(\pi)}\lambda_{i\pi(i)}\cdots\lambda_{i\pi(i)}\cdots\lambda_{n\pi(n)}\Bigg) \\
& =c\cdot\sum_{\pi\in A_n}(\sgn(\pi)-\sgn(\pi)) \cdot \cdots \\
& \cdot \cdots \lambda_{i\pi(i)}\cdots\lambda_{i\pi(i)}\cdots\lambda_{n\pi(n)}=0
\end{aligned}
\]
\end{enumerate}
\end{proof}
\end{satz}
\subsubsection{Bemerkung}
Es gibt also zu jedem $\mathbb{K}$-VR V mit $\dim(V)=n$ eine nicht ausgeartete alternierende n-Linearform.
\begin{satz} \label{theo:1.2.6}
Sei V $\mathbb{K}$-VR mit $\dim(V)=n$ und $\varphi_1, \varphi_2$ nicht ausgeartete alternierende n-Linearformen. Dann existiert $c\in\mathbb{K}\setminus\{0\}$ mit $\varphi_2=c\cdot\varphi_1$.
\begin{proof}
Sei $a_1, \dots, a_n$ Basis von V. Nach Lemma \ref{theo:1.2.4} ist
$\varphi_i(a_1, \dots, a_n)\neq0, i=1, 2$.\\
Sei $c:=\dfrac{\varphi_1(a_1, \dots, a_n)}{\varphi_2(a_1, \dots, a_n)} \in \mathbb{K}\setminus\{0\}$.\\
Sei $b_1, \dots, b_n$ mit $b_i=\sum\lambda_{ij}a_j$.\\
Dann gilt nach Satz \ref{theo:1.2.5}(a), dass für $i=1, 2$
\[\begin{aligned}
& \varphi_i(b_1, \dots, b_n) = \varphi_i(a_1, \dots, a_n)\underbrace{\sum_{\pi \in S_n}\lambda_{1\pi(1)}\cdots\lambda_{n\pi(n)}}_{\text{unabhängig von $i$!}} \\
& \implies \frac{\varphi_1(b_1, \dots, b_n)}{\varphi_2(b_1, \dots, b_n)}=\frac{\varphi_1(a_1, \dots, a_n)}{\varphi_2(a_1, \dots, a_n)}=c
\end{aligned}
\]
\end{proof}
\end{satz}
\section{Determinanten}
\begin{defin}
Sei $B=(a_1, \dots, a_n)$ Basis des $\mathbb{K}$-Vektorraums V.
Sei $\varphi$ nicht ausgeartete n-Linearform und $\alpha \in \homk$. Dann ist die \underline{Determinante von $\alpha$} definiert durch \[\det(\alpha):=\det{}_\mathbb{K}(\alpha):=\frac{\varphi(\alpha(a_1), \dots, \alpha(a_n))}{\varphi(a_1, \dots, a_n)}\]
\end{defin}
\begin{satz} \label{theo:1.3.2}
$\det(\alpha)$ ist unabhängig von der Wahl der Basis B und der der Form $\varphi$.
\begin{proof}
\begin{itemize}
\item[1. Fall] $\alpha$ nicht bijektiv\\ $\implies \alpha(a_1), \dots, \alpha(a_n) \text{linear unabhängig} \implies \det(\alpha) = 0$
\item[2. Fall] $\alpha$ bijektiv. Sei $B=(a_1, \dots, a_n)$.
Dann ist auch $\alpha(a_1), \dots, \alpha(a_n)$ Basis und,
da $\varphi$ nicht ausgeartet,
\[\varphi(\alpha(a_1), \dots, \alpha(a_n))\neq0\]
Sei $\varphi_\alpha(b_1, \dots, b_n) := \varphi(\alpha(b_1), \dots, \alpha(b_n))$.
Dann ist $\varphi_\alpha$ alternierend und nicht ausgeartet. Wegen Satz \ref{theo:1.2.6} folgt, dass $c\in\mathbb{K}\setminus\{0\}$ existiert mit
\begin{equation}\label{eq:constantphi}
\varphi_\alpha=c\cdot\varphi
\end{equation}
und (durch Einsetzen von $a_1, \dots, a_n$), dass $c=\det(\alpha)$. Da \ref{eq:constantphi} unabhängig von B ist also $\det(\alpha)$ unabhängig von B.
Sei nun $\psi$ eine zweite alternierende, nicht ausgeartete n-Form und $\psi_\alpha(b_1, \dots, b_n) := \psi(\alpha(b_1), \dots, \alpha(b_n))$. Dann ist $\psi_\alpha$ alternierend und nicht ausgeartet. Nach Satz \ref{theo:1.2.6} gibt es $d\in\mathbb{K}\setminus\{0\} \text{ mit }d=\frac\psi\varphi$.
Also gilt:
\[
\det(\alpha)=\frac{\varphi_\alpha(a_1, \dots, a_n)}{\varphi(a_1, \dots, a_n)}=\frac{d\varphi_\alpha(a_1, \dots, a_n)}{d\varphi(a_1, \dots, a_n)}=\frac{\psi_\alpha(a_1, \dots, a_n)}{\psi(a_1, \dots, a_n)}
\]
also ist $\det(\alpha)$ auch von der n-Form unabhängig.
\end{itemize}
\end{proof}
\end{satz}
\begin{korollar} \label{theo:1.3.3}
Sei V ein n-dimensionaler $\mathbb{K}$-Vektorraum. Dann gilt
\begin{enumerate}[label=\alph*)]
\item $\alpha\in \homk \text{ bijektiv } \iff \det(\alpha)\neq0$
\item $\alpha, \beta \in \homk \implies \det(\alpha, \beta) = \det(\alpha) \det(\beta)$
\item $\det(\id)=1$
\item Ist $\alpha\in \homk$ invertierbar, so gilt $\det(\alpha^{-1})=\det(\alpha)^{-1}$.
\end{enumerate}
\begin{proof}
Sei $B=(a_1, \dots, a_n)$ Basis und $\varphi$ n-Form mit \[
\det(\alpha) = \frac{\varphi(\alpha(a_1), \dots, \alpha(a_n))}{\varphi(a_1, \dots, a_n)}\text{[unabhängig von $B$ und $\varphi$ nach Satz \ref{theo:1.3.2}]}
\]
\begin{enumerate}[label=\alph*)]
\item $\alpha$ bijektiv $\iff \alpha(a_1), \dots, \alpha(a_n) \text{ linear unabhängig}$\\
$\underbrace{\iff}_{\text{Lemma \ref{theo:1.2.4}}}
\varphi(\alpha(a_1), \dots, \alpha(a_n))\neq0\iff \det(\alpha)\neq0$
\item 2 Fälle:\begin{enumerate}[label=\arabic. Fall]
\item[1. Fall:] $\alpha$ oder $\beta$ ist nicht bijektiv: o.B.d.A $\alpha$ nicht bijektiv.\\
$\implies \det(\alpha)=0\implies \det(\alpha)\det(\alpha)=0$\\
Weiters folgt, dass $\alpha\beta$ nicht bijektiv, also $\det(\alpha\beta)=0$.
\item[2. Fall:] $\alpha, \beta$ bijektiv. Dann ist auch $(\beta(a_1), \dots, \beta(a_n))$ Basis und
\[
\begin{aligned}
\det(\alpha\beta) & = \frac{\varphi(\alpha(\beta(a_1)), \dots, \alpha(\beta(a_n)))}{\varphi(a_1, \dots, a_n)} \\
& =\frac{\varphi(\alpha(\beta(a_1)), \dots, \alpha(\beta(a_n)))}{\varphi(\beta(a_1), \dots, \beta(a_n))}\cdot \cdots \\
& \cdots \frac{\varphi(\beta(a_1), \dots, \beta(a_n))}{\varphi(a_1, \dots, a_n)}\underbrace{=}_{\text{Satz \ref{theo:1.3.2}}} \det(\alpha)\det(\beta)
\end{aligned}
\]
\end{enumerate}
\item $\det(\id)=\frac{\varphi(a_1, \dots, a_n)}{\varphi(a_1, \dots, a_n)}=1$
\item $1\underbrace{=}_{\text{c)}}\det(\id)=\det(\alpha\alpha^{-1})\underbrace{=}_{\text{b)}}\det(\alpha)\det(\alpha^{-1})$
\end{enumerate}
\end{proof}
\end{korollar}
\begin{satz} \label{theo:1.3.4}
Sei $\alpha\in \homk, B=(b_1, \dots, b_n)$ Basis und $A=(a_{ij}) = {}_B M(\alpha)_B\in\mathbb{K}^{n\times n}$. Dann gilt
\[\det(\alpha)=\sum_{\pi\in S_n}\sgn(\pi)a_{1\pi(1)}\cdots a_{n\pi(n)}\]
\begin{proof}
Es gilt $\alpha(b_i)=\sum\limits_{j=1}^na_{ij}b_j \text{ für }i=1, \dots, n$.
Nach Satz \ref{theo:1.2.5}(a) gilt
\[
\varphi(\alpha(b_1), \dots, \alpha(b_n)) = \varphi(b_1, \dots, b_n)\cdot\sum_{\pi\in S_n}\sgn(\pi)a_{1\pi(1)}\cdots a_{n\pi(n)}
\]
und daraus folgt die Behauptung direkt.
\end{proof}
\end{satz}
\begin{defin}
Für $A=(a_{ij})\in\mathbb{K}^{n\times n}$ definieren wir die \underline{Determinante von A} als
\[
\det(A)=\sum_{\pi\in S_n} \sgn(\pi)a_{1\pi(1)}\cdots a_{n\pi(n)}\in\mathbb{K}
\]
\end{defin}
\subsubsection{Bemerkung}
Schreibweise für $A=(a_{ij})$:
\[
\det(A)=\begin{vmatrix}
a_{11} & \dots & a_{1n} \\
\vdots & \ddots & \vdots \\
a_{n1} & \dots & a_{nn}
\end{vmatrix}
\]
\section{Rechenregeln}
\begin{satz}\label{theo:1.4.1}
Sei $A=(a_1, \dots, a_n)\in\mathbb{K}^{n\times n}$. Dann gilt
\begin{enumerate}[label=\alph*)]
\item $\det(A)=\det(A^T)$
\item $\forall i, j\in[n]: i<n: \det((a_1, \dots, \underbrace{a_j}_{i}, \dots, \underbrace{a_i}_{j}, \dots, a_n))=\det(A)$
\item $\forall i\in[n], \lambda_1, \dots, \lambda_n\in\mathbb{K}: \det((a_1, \dots, a_i+\sum\limits_{\substack{j=1\\j\neq i}}^n\lambda_ja_j, \dots, a_n))=\det(A)$
\item $\forall i\in[n], \lambda\in\mathbb{K}: \det((a_1, \dots, \lambda a_i, \dots, a_n)) = \det(A)$
\item $\exists i, j\in[n]: i\neq j\land a_i=a_j \implies \det(A)=0$
\item $\forall \lambda \in \mathbb{K}: \det(\lambda A)=\lambda^n \det(A)$
\item $A$ invertierbar $\implies \det(A^{-1})=\det(A)^{-1}$
\item $\forall B \in \mathbb{K}^{n\times n}: \det(AB)=\det(A)\det(B)$
\item $\det(I_n)=1$
\end{enumerate}
\begin{proof}
Nur a) explizit:
\begin{enumerate}[label=\alph*)]
\item \begin{equation*}\begin{aligned}
\det(A^T) & = \sum_{\pi\in S_n}\sgn(\pi)a_{\pi(1)1}\cdots a_{\pi(n)n} \\
& =\sum_{\pi\in S_n}\sgn(\pi)a_{1\pi^{-1}(1)}\cdots a_{n\pi^{-1}(n)} \\
& \underbrace{=}_{\substack{\sgn(\pi^{-1})=\sgn(\pi) \\\pi^{-1}\mapsto\pi}} \sum_{\pi\in S_n} \sgn(\pi)a_{1\pi(1)}\cdots a_{n\pi(n)}
\end{aligned}\end{equation*}
\item[b) - i)] folgt daraus, dass für \[\alpha:\begin{cases}\mathbb{K}^n\to\mathbb{K}^n\\x\mapsto A\cdot x\end{cases}:
\det(A)=\dfrac{a}{b}\text{ (Satz \ref{theo:1.3.4})}\] und, dass $\varphi$ alternierende n-Form ist, bzw. Korollar \ref{theo:1.3.3}
\end{enumerate}
\end{proof}
\end{satz}
\begin{satz}
Seien $A, B\in\mathbb{K}^{n\times n}$ ähnlich,
das heißt $\exists P\in\mathbb{K}^{n\times n}$ invertierbar mit \\
$B=P^{-1}\cdot A\cdot P$. Dann gilt
\[\det(A)=\det(B)\]
Weiters ist A genau dann invertierbar wenn $\det(A)\neq0$.
\begin{proof}
\[\det(B)=\det(P)\underbrace{\det(P^{-1})}_{=\det(P)^{-1}}\det(A)=\det(A)\]
Rest folgt, da $\det(A)=\det(\alpha)$ mit $\alpha:\begin{cases}\mathbb{K}^n\to\mathbb{K}^n \\x\mapsto A\cdot x
\end{cases}$.
\end{proof}
\end{satz}
\subsubsection{Berechnungsverfahren}
Gaußalgorithmus führt 1) Zeilenvertauschungen und 2) Additionen von\\
Vielfachen einer Zeile zu einer anderen durch. Raus kommt eine obere Dreiecksmatrix.
\begin{equation}\label{eq:dreiecksmatrix}
B=\begin{pmatrix}
b_{11} & \dots & \dots & b_{1n} \\
0 & b_{22} & \dots & b_{2n} \\
\vdots & & \ddots & \vdots \\
0 & \dots & \dots & b_{nn}
\end{pmatrix}
\end{equation}
Operationen 2) ändern die Determinante nicht, Operationen 1) ändern das Vorzeichen.
\begin{satz}
Sei $A\in\mathbb{K}^{n\times n}$ und $B$ wie \ref{eq:dreiecksmatrix} das Resultat des Gaußalgorithmus auf $A$ angewendet mit $k$ Zeilenvertauschungen. Dann gilt
\[
\det(A)=(-1)^kb_{11}\cdot\dots\cdot b_{nn}
\]
\begin{proof}
Für Matrizen der Form \ref{eq:dreiecksmatrix} ist die Determinante das Produkt der Diagonalelemente. Rest folgt aus der Definition des Gaußalgorithmus, sowie Satz \ref{theo:1.4.1}.
\end{proof}
\end{satz}
\subsubsection{Regel von Sarrus}
Sei $A=\begin{pmatrix}
a_{11} & a_{12} & a_{13} \\
a_{21} & a_{22} & a_{23} \\
a_{31} & a_{32} & a_{33}
\end{pmatrix}\in\K^{3\times3}$
\[\begin{array}{ccccccccc}
a_{11} & & a_{12} & & a_{13} & & a_{11} & & a_{12} \\
& \color{ForestGreen}\diagdown & & \color{ForestGreen}\diagdown \color{red} \mathllap \diagup & & \color{ForestGreen}\diagdown \color{red} \mathllap \diagup & & \color{red}\diagup \\
a_{21} & & a_{22} & & a_{23} & & a_{21} & & a_{22} \\
& \color{red} \diagup & & \color{ForestGreen}\diagdown \color{red} \mathllap \diagup & & \color{ForestGreen}\diagdown \color{red} \mathllap \diagup & & \color{ForestGreen}\diagdown \\
a_{31} & & a_{32} & & a_{33} & & a_{31} & & a_{32}
\end{array} \color{ForestGreen} + + + \color{red} - - -\]
$A=\begin{pmatrix}a_{11} & a_{12} \\ a_{21} & a_{22}\end{pmatrix}\in\K^{2\times2} \implies \det(A)=a_{11}a_{22}-a_{12}a_{21}$\\
$n>3 \to $ Gaußalgorithmus
\begin{defin}
Sei $A\in\mathbb{K}^{n\times n}$ und $i, j\in[n]$. Sei $M_{ij}\in\mathbb{K}^{n\times n}$ die Matrix, welche durch Ersetzung der j-ten Spalte durch den i-ten Einheitsvektor $e_j$ entsteht.\\
$A_{ij}:=\det(M_{ij})$ heißt \underline{Kofaktor} (zum Indexpaar $(i, j)$).
\begin{equation*}
\bordermatrix{
&&&&j \cr
&a_{11}&\dots &a_{1i-1}&0&a_{1i+1}&\dots&a_{1n} \cr
&\vdots&\ddots&\vdots&\vdots&\vdots&\ddots&\vdots \cr
i&a_{ji}&\dots&a_{ji-1}&1&a_{ji+1}&\dots&a_{jn}\cr
&\vdots&\ddots&\vdots&\vdots&\vdots&\ddots&\vdots \cr
&a_{n1}&\dots &a_{ni-1}&0&a_{ni+1}&\dots&a_{nn}
}
= M_{ij}=(a_{\_1}, \dots, \underbrace{e_i}_{j}, \dots, a_{\_n})
\end{equation*}
\end{defin}
\subsubsection{Bemerkung}
Es gilt
\begin{equation}\label{crazymatrix}
A_{ij}=\begin{vmatrix}
a_{11} & \dots & a_{1i-1} & 0 & a_{1i+1} & \dots & a_{1n} \\
\vdots & \ddots & \vdots & \vdots & \vdots & \ddots & \vdots \\
a_{ji} & \dots & a_{ji-1} & 1 & a_{ji+1} & \dots & a_{jn} \\
\vdots & \ddots & \vdots & \vdots & \vdots & \ddots & \vdots \\
a_{n1} & \dots & a_{ni-1} & 0 & a_{ni+1} & \dots & a_{nn}
\end{vmatrix}
\end{equation}
da obige Matrix aus $M_{ij}$ durch Spaltenadditionen hervorgeht.
\begin{lemma}
Sei $\tilde{A_{ij}}\in\mathbb{K}^{(n-1)\times(n-1)}$ die Matrix, welche aus A durch Streichung der i-ten Spalte und j-ten Zeile hervorgeht und $D_{ij}:=\det(\tilde{A_{ij}})$. Dann gilt \[A_{ij}=(-1)^{i+j}D_{ij}\]
\begin{proof}
Transformiere durch ($i-1$) Spaltenvertauschungen und ($j-1$) Zeilenvertauschungen die Matrix \ref{crazymatrix} auf
\[
B_{ij} = \begin{pmatrix}
1 & 0 & \dots & 0 \\
0 & & & \\
\vdots & & \tilde{A_{ij}} & \\
0 & & &
\end{pmatrix}
\]
Es gilt $\lvert B_{ij}\rvert=D_{ij}$ und $\lvert B_{ij}\rvert=(-1)^{(i-1)+j(-1)}A_{ij}$ woraus die Behauptung folgt.
\end{proof}
\end{lemma}
\begin{satz}[Entwicklungssatz von Laplace]
Sei $A\in\mathbb{K}^{n\times n}$ und $i, j\in[n]$. Dann gilt
\begin{enumerate}[label=\alph*)]
\item $\det(A) = \sum\limits_{l=1}^na_{il}A_{il} = \sum\limits_{l=1}^n(-1)^{l+i}a_{il}D_{il}$
\item $\det(A) = \sum\limits_{l=1}^na_{lj}A_{lj} = \sum\limits_{l=1}^n(-1)^{l+j}a_{lj}D_{lj}$
\end{enumerate}
\begin{proof}
b) \[\begin{aligned}
\det(A) & = \det(a_{\_1}, \dots, a_{\_n})= \\
& =\det(a_{\_1}, \dots, \underbrace{\sum_{l=1}^na_{lj}e_l}_{=a_{\_j}}, \dots, a_{\_n})= \\
& =\sum_{l=1}^n a_{lj}\det(a_{\_1}, \dots, \underbrace{e_l}_{j}, \dots, a_{\_n}) = \\
& = \sum_{l=1}^n a_{lj}A_{lj}
\end{aligned}
\]
a) analog (angewendet auf $A^T$).
\end{proof}
\end{satz}
\begin{satz}[Cramer'sche Regel]
Sei $\adj(A)=(A_{ji})_{i, j\in[n]}$. Dann gilt
\[A\cdot \adj(A) = \det(A)\cdot I_n\]
\begin{proof}
Sei $B=A\cdot\adj(A)\implies$
\[\begin{aligned}
b_{ij} & = \sum_{k=1}^n a_{ik} A_{jk} \\
& = \sum_{k=1}^n a_{ik} \det(a_{\_1}, \dots, \underbrace{e_j}_{k}, \dots, a_{\_n}) \\
& = \sum_{k=1}^n a_{ik}
\bordermatrix{
& & & k & & \\
& a_{11} & \dots & a_{1k} & \dots & a_{1n} \\
& \vdots & \ddots & \vdots & \ddots & \vdots \\
j & 0 & \dots & 1 & \dots & 0 \\
& \vdots & \ddots & \vdots & \ddots & \vdots \\
& a_{n1} & \dots & a_{nk} & \dots & a_{nn} \\
} \\
& = \det\left(\bordermatrix{ & \\& a_{1\_} \\ & \vdots \\ j \to & a_{i\_} \\ & \vdots \\ & a_{n\_}}\right) \\
& = \begin{cases}0& i\neq j \\ \det(A) & i=j\end{cases}
\end{aligned}\]
\end{proof}
\end{satz}
\begin{folgerung}
Sei $A\in\mathbb{K}^{n\times n}$ invertierbar. Sei $x\in\mathbb{K}^n$ die eindeutige Lösung des linearen Gleichunssystems $Ax=b$. Dann gilt
\[
x_i= \det(A)^{-1} \det(a_{\_1}, \dots, \underbrace{b}_{i}, \dots, a_{\_n})
\]
\begin{proof}
\[\begin{aligned}
& A^{-1}=\frac{1}{\det(A)}(A_{ji}) \\
& \implies \det(A)x_i=\sum_{j=1}^n A_{ji}b_j & = \sum_{j=1}^n b_j \det(a_{\_1}, \dots, \underbrace{e_j}_{i}, \dots, a_{\_n}) \\
& & =\det(a_{\_1}, \dots, \underbrace{b}_{i}, \dots, a_{\_n})
\end{aligned}\]
\end{proof}
\end{folgerung}
\subsubsection{Blockmatrizen}
\begin{defin}
$A\in\mathbb{K}^{n\times n}$ heißt \underline{obere Blockmatrix} wenn $\exists p\in \{1, \dots, n-1\}$ mit $a_{ij}=0$ für $p+1\le i\le n, 1\le j\le p$, d.h.
\begin{equation}
\label{blockmatrix}
A=\bordermatrix{
\ &\overbrace{}^{p} & \overbrace{}^{n-p} \cr
p\{\ & P & D \cr % } TODO geschwungene Klammern besser machen
n-p\{\ &0&Q} % }
\end{equation}
Analog sind \underline{untere Blockmatrizen} definiert.
\end{defin}
\begin{satz} \label{theo:1.4.10}
Sei $A$ obere Blockmatrix wie in \ref{blockmatrix}. Dann gilt $\det(A)= \det(P) \det(Q)$
\begin{proof}
Sei $A = \begin{pmatrix} P & D \\ 0 & Q \end{pmatrix}$.\\
Wende elementare Zeilenumformungen der ersten $p$ Zeilen an, sodass $P$ obere Dreiecksform hat (mit $s$ Zeilenvertauschungen) und elementare Zeilenumformungen der letzten $n-p$ Zeilen sodass $Q$ obere Dreiecksform hat (mit $t$ Zeilenvertauschungen). Bezeichne das Ergebnis mit $A'= \begin{pmatrix} P' & D \\ 0 & Q' \end{pmatrix}$, wobei $P', Q'$ obere Dreiecksform haben.\\
Es folgt, dass $A', P', Q'$ obere Dreiecksform hat. Da die Determinante oberer Dreiecksmatrizen das Produkt der Diagonalelemente ist, gilt $\det(A')=\det(P')\det(Q')$.\\
Weiters gilt $\det(A')=(-1)^{s+t} \det(A)$ (insgesamt $s+t$ Vertauschungen) und $\det(P')= (-1)^s \det(P), \det(Q') = (-1)^t \det(Q)$. Daraus folgt die Behauptung.
\end{proof}
\end{satz}
\chapter{Eigenwerte und Eigenvektoren}
\section{Diagonalisierbarkeit}
\begin{defin}
$D\in \mathbb{K}^{n\times n}$ heißt \underline{Diagonalmatrix} wenn $\forall i\neq j: d_{ij}=0$.
Wir schreiben auch
\[
\diag(\lambda_1, \dots, \lambda_n):=\begin{pmatrix}
\lambda_1 & 0 & \dots & 0 \\
0 & \lambda_2 & \dots & 0 \\
\vdots & \vdots & \ddots & \vdots \\
0 & 0 & \dots & \lambda_n
\end{pmatrix}
\]
\end{defin}
\subsubsection{Bemerkung}
\begin{itemize}
\item $A\in \mathbb{K}^{n\times m} \implies \diag(\lambda_1, \dots, \lambda_n)A = \begin{pmatrix}
\lambda_1 a_{1\_} \\
\vdots \\
\lambda_n a_{n\_}\end{pmatrix}$
\item $\diag(\lambda_1, \dots, \lambda_n)^k = \diag(\lambda_1^k, \dots, \lambda_n^k)$
\end{itemize}
\begin{defin}
\leavevmode
\begin{enumerate}[label=\alph*)]
\item $\alpha \in \homk, \dim(V)<\infty$ heißt \underline{diagonalisierbar} (bzgl. $B$)
wenn eine geordnete Basis $B$ existiert mit ${}_B M(\alpha)_B$ Diagonalmatrix
\item $A\in\mathbb{K}^{n\times n}$ heißt diagonalisierbar wenn eine invertierbare Matrix $P\in\mathbb{K}^{n\times n}$ existiert mit $P^{-1}AP$ Diagonalmatrix.
\end{enumerate}
\end{defin}
\begin{lemma}
Sei $V$ $\K$-Vektorraum mit $\dim(V)=n<\infty$.
Dann gilt für $\alpha\in\homk$ und $C$ Basis:
\[\alpha \text{ diagonalisierbar} \iff {}_C M(\alpha)_C \text{ diagonalisierbar}\]
\begin{proof}
\begin{itemize}
\item[$\implies$] Sei $\alpha$ diagonalisierbar und $B$ eine Basis mit $_B M(\alpha)_B$ Diagonalmatrix. Dann gilt \[
\begin{aligned} {}_B M(\alpha)_B & = {}_B M(\id)_C \cdot {}_C M(\alpha)_C \cdot {}_C M(\id)_B \\
& = {}_C M(\id)_B^{-1} \cdot {}_C M(\alpha)_C \cdot {}_C M(\id)_B\end{aligned}\]
Also ist ${}_C M(\alpha)_C$ diagonalisierbar.
\item[$\impliedby$] Sei ${}_C M(\alpha)_C$ diagonalisierbar und $P$ invertierbar mit $P^{-1}\cdot {}_C M(\alpha)_C \cdot P$ Diagonalmatrix. Sei $B$ Basis mit $P={}_C M(\id)_B$. Dann gilt ${}_B M(\alpha)_B$ ist Diagonalmatrix.
\end{itemize}
\end{proof}
\end{lemma}
\begin{lemma} \label{theo:2.1.4}
\leavevmode
\begin{enumerate}[label=\alph*)]
\item $\alpha \in \homk$ ist diagonalisierbar genau wenn es eine Basis $B=(b_1, \dots, b_n)$ und $\lambda_1, \dots, \lambda_n\in\K$ gibt mit $\forall i=1, \dots, n:\alpha(b_i)=\lambda_i b_i$.
\item $A\in\K^{n\times n}$ ist diagonalisierbar genau wenn es eine geordnete Basis $B= (b_1, \dots, b_n)$ von $\K^n$ gibt mit $\forall i=1, \dots, n: A b_i = \lambda_i b_i$.
\end{enumerate}
\begin{proof}
\begin{enumerate} [label=\alph*)]
\item die Bedingung ist äquivalent zu ${}_B M(\alpha)_B$ diagonalisierbar.
\item Spezialfall von a).
\end{enumerate}
\end{proof}
\end{lemma}
\section{Eigenwerte und Eigenvektoren}
\begin{defin} \label{theo:2.2.1}
\leavevmode
\begin{enumerate}[label=\alph*)]
\item Sei $\alpha \in \homk$. $\lambda\in\K$ heißt \underline{Eigenwert} von $\alpha$ wenn es einen Vektor $v\in V\setminus\{0\}$ gibt mit $\alpha(v)=\lambda v$. $v$ heißt \underline{Eigenvektor} zu $\lambda$.\\
Die Menge aller Eigenwerte von $\alpha$ heißt \underline{Spektrum} von $\alpha; \spec(\alpha)$
\item Sei $A \in \K^{n\times n}$. $\lambda\in\K$ heißt \underline{Eigenwert} von $A$ wenn es $v\in \K^n\setminus\{0\}$ gibt mit $A v = \lambda v$. $v$ heißt \underline{Eigenvektor} zu $\lambda$.\\
Die Menge aller Eigenwerte von $A$ heißt \underline{Spektrum} von $A; \spec(A)$
\end{enumerate}
\end{defin}
\begin{lemma} \label{theo:2.2.2}
\leavevmode
\begin{enumerate}[label=\alph*)]
\item $\alpha \in \homk$ diagonalisierbar $\iff \exists$ Basis aus Eigenvektoren.
\item $A \in \K^{n\times n}$ diagonalisierbar $\iff \exists$ Basis aus Eigenvektoren.
\end{enumerate}
\begin{proof}
Folgt direkt aus Lemma \ref{theo:2.1.4} und Definition \ref{theo:2.2.1}
\end{proof}
\end{lemma}
\begin{defin}
\leavevmode
\begin{enumerate}[label=\alph*)]
\item Sei $\alpha \in \homk$ und $\lambda \in \spec(\alpha)$. Dann heißt $\eig_\alpha(\lambda):=\{v\in V: \alpha(v) = \lambda v \}$ der zugehörige \underline{Eigenraum}.
\item Sei $A \in \K^{n\times n}$ und $\lambda \in \spec(A)$. Dann heißt $\eig_A(\lambda):=\{v\in \K^n: A v = \lambda v \}$ der zugehörige \underline{Eigenraum}.
\end{enumerate}
\end{defin}
\begin{lemma}
Sei $\alpha \in \homk / A\in\K^{n\times n}$ und $\lambda \in \spec(\alpha)/\lambda\in\spec(A)$.\\
Dann ist $\eig_\alpha(\lambda)/\eig_A(\lambda)$ ein Unterraum von $V/\K$.
\begin{proof}
Nur für $\alpha\in\homk$
\begin{itemize}
\item $ 0 = \alpha(0) = \lambda \cdot 0 \implies 0 \in \eig_\alpha(\lambda) $
\item $v, w\in \eig_\alpha(\lambda) \implies \alpha(v+w) = \alpha(v) + \alpha(w) = \lambda v + \lambda w = \lambda(v + w) \implies v + w \in \eig_\alpha(V)$
\item $\mu \in \K, v \in \eig_\alpha(\lambda) \implies \alpha(\mu v) = \mu \cdot \alpha(v) = \mu \cdot \lambda \cdot v = \lambda \cdot (\mu \cdot v) \implies \mu \cdot v \in \eig_\alpha(\lambda)$
\end{itemize}
\end{proof}
\end{lemma}
\begin{satz}
Sei $\alpha \in \homk$ und $B$ Basis. Dann gilt
\[\begin{aligned}
& \spec(\alpha) = \spec({}_B M(\alpha)_B) \\
& {}_B\Phi(\eig_\alpha(\lambda)) = \eig_{{}_B M(\alpha)_B}(\lambda)
\end{aligned}\]
\begin{proof}
Sei $\lambda \in \spec(\alpha)$ und $v\in\eig_\alpha(\lambda)$. Dann gilt \[
\alpha(v) = \lambda v \iff {}_B M(\alpha)_B \cdot {}_B v = \lambda \cdot {}_B v
\]
\end{proof}
\end{satz}
\begin{defin}
\leavevmode
\begin{enumerate}[label=\alph*)]
\item Sei $\alpha \in \homk, \dim(V)<\infty$ und $B$ Basis. Dann heißt die Funktion \[
\chi_\alpha:\begin{cases}\K \to \K \\
\lambda \mapsto \det({}_B M(\alpha)_B - \lambda \cdot I_n)\end{cases}
\] \underline{charakteristisches Polynom} von $\alpha$.
\item Sei $A \in \K^{n\times n}$. Dann heißt die Funktion \[
\chi_A:\begin{cases}\K \to \K \\
\lambda \mapsto \det(A - \lambda \cdot I_n)\end{cases}
\] \underline{charakteristisches Polynom} von $A$.
\end{enumerate}
\end{defin}
\subsubsection{Bemerkung}
$\genfrac{}{}{0pt}{0}{\chi_\alpha}{\chi_A}$ ist Polynom vom Grad
$\le\genfrac{}{}{0pt}{0}{\dim(V)}{n}$, da
\[\begin{aligned}
& \chi_A(\lambda)=\sum_{\pi \in S_n} \tilde{a}_{1\pi(1)}^{(\lambda)} \cdots \tilde{a}_{n\pi(n)}^{(\lambda)} \text{ mit} \\
& \tilde{a}_{ij}^{(\lambda)} = \begin{cases} a_{ij} & i\neq j \\ a_{ij}-\lambda & i=j
\end{cases} \dots \text{ Polynom von Grad $0$ oder $1$}
\end{aligned}\]
\begin{lemma} \label{theo:2.2.7}
\leavevmode
\begin{enumerate}[label=\alph*)]
\item $\chi_\alpha$ ist unabhängig von der Wahl der Basis.
\item $\chi_A = \chi_B$ wenn $A, B$ ähnlich (das heißt $\exists P \in \K^{n \times n}: B = P^{-1}AP$)
\end{enumerate}
\begin{proof}
\begin{enumerate}[label=\alph*)]
\item Sei C weitere Basis.\\
Dann gilt $\underbrace{{}_C M(\alpha)_C}_{B} = \underbrace{{}_C M(\id)_B}_{P^{-1}} \underbrace{{}_B M(\alpha)_B}_{A} \underbrace{{}_B M(\id)_C}_{P}$. \\
Man kann also alles auf b) zurückführen.
\item \[\begin{aligned}
\chi_A(\lambda) & = \det(A-\lambda I) \\
& = \det(P)^{-1} \det(A - \lambda I) \det(P) \\
& = \det(P^{-1}) \det(A - \lambda I) \det(P) \\
& = \det(P^{-1}(A - \lambda I)P) \\
& = \det(P^{-1}AP-\lambda I) \\
& = \det(B - \lambda I) \\
& = \chi_B(\lambda)
\end{aligned}\]
\end{enumerate}
\end{proof}
\end{lemma}
\begin{lemma}
\leavevmode
\begin{enumerate}[label=\alph*)]
\item Sei $\alpha\in\homk$. Dann gilt \[\spec(\alpha) = \{\lambda \in \K: \chi_\alpha(\lambda)=0\}\]
\item Sei $A\in \K^{\nxn}$. Dann gilt \[\spec(A) = \{\lambda \in \K: \chi_A(\lambda)=0\}\]
\end{enumerate}
\begin{proof}
Nur b)
\[\begin{aligned}
\lambda \in \spec(A) & \iff \exists v\in V \setminus \{0\}: A v = \lambda v \\
& \iff \exists v \in V \setminus \{0\}: (A - \lambda I) v = 0 \\
& \iff \ker(A - \lambda I) \neq \{0\} \\
& \iff A - \lambda I \text{ nicht injektiv} \\
& \iff \det(A - \lambda I) = 0
\end{aligned}\]
\end{proof}
\end{lemma}
\subsubsection{Beispiele}
\begin{alignat*}{3}
& A = \begin{pmatrix}\bar3 & \bar4 \\ \bar1 & \bar1 \end{pmatrix} \in \mathbb{Z}_5^{2\times2} & & \\
& \chi_A(\lambda) = \begin{vmatrix} \bar3 - \lambda & \bar4 \\ \bar1 & \bar1 - \lambda \end{vmatrix}
& & = (\bar3 - \lambda)(\bar1 - \lambda) - \bar4 \\
& & & = \bar3 - \bar4 \lambda + \lambda^2 - \bar4 \\
& & & = \bar4 - \bar4 \lambda + \lambda^2 = (\bar2 - \lambda)^2 \\
& \implies \spec(A) = \{2\} \\
& \eig_{\bar2}(A) = ? \\
& v \in \eig_{\bar2}(A) \iff \mathrlap{(A - \bar2 I)v = 0} \\
& \iff \mathrlap{\left(\begin{array}{c c | c}
\bar3 - \bar2 & \bar4 & \bar0 \\
\bar1 & \bar1 - \bar2 & \bar0
\end{array}\right)} \\
& \left(\begin{array}{c c | c}
\bar1 & \bar4 & \bar0 \\
\bar1 & \bar4 & \bar0
\end{array}\right) \\
& \left(\begin{array}{c c | c}
\bar1 & \bar4 & \bar0 \\
\bar0 & \bar0 & \bar0
\end{array}\right) \\
& \implies \eig_{\bar2}(A) = \bigg\langle\begin{pmatrix}\bar1 \\ \bar1\end{pmatrix} \bigg\rangle \\
& \implies A \mathrlap{\text{ nicht diagonalisierbar [Lemma \ref{theo:2.1.4} (b)]}}
\end{alignat*}
\begin{lemma}
Sei $A \in \mathbb{C}^{n\times n}$ mit reellen Einträgen. Dann gilt:
\begin{enumerate}[label=\alph*)]
\item $\lambda \in \spec(A) \implies \overline{\lambda} \in \spec(A)$
\item $v \in \eig_\lambda(A) \implies \overline{v} \in \eig_{\overline{\lambda}}(A)$
\end{enumerate}
\begin{proof}
\begin{enumerate}[label=\alph*)]
\item Klarerweise ist $\chi_A(\lambda)$ ein Polynom mit reellen Koeffizienten, also $\chi_A(\lambda)=a_0+a_1 \lambda + \cdots + a_n \lambda^n, a_0, \dots, a_n\in\mathbb{R}$\\
Sei $\chi_A(\lambda)=0 \implies 0 = \overline0 = a_0 + a_1 \overline\lambda + \cdots + a_n \overline{\lambda} ^ n = \chi_A(\overline\lambda)$
\item $v\in\eig_\lambda(A) \implies A v = \lambda v \implies \overline{A V} = \overline{\lambda v} \implies A \overline{v} = \overline\lambda \overline{v}$
\end{enumerate}
\end{proof}
\end{lemma}
\begin{lemma} \label{theo:2.2.10}
Eigenvektoren zu unterschiedlichen Eigenwerten sind linear unabhängig.
\begin{proof}
Seien $v_i \in \eig_{\lambda_i}(A), i=1, \dots, r, \lambda_i \neq \lambda_j \text{ für } i\neq j$.
Induktion nach $r$
\begin{itemize}
\item[$r=1$:] $v_1$ ist linear unabhängig.
\item[$r-1\mapsto r$:] \begin{equation}\label{eq:2.2.10.1}
\mu_1 v_1 + \cdots + \mu_1 v_1 = 0 \end{equation}
\[ \implies A(\mu_1 v_1 + \cdots + \mu_r v_r) = 0 \]
\begin{equation}\label{eq:2.2.10.2}
\implies \lambda_1\mu_1 v_1 + \cdots \lambda_r \mu_r v_r = 0
\end{equation}
Weiters folgt durch Multiplikation von \ref{eq:2.2.10.1} mit $\lambda_r$, dass \begin{equation}\label{eq:2.2.10.3}
\lambda_r \mu_1 v_1 + \cdots + \lambda_r \mu_r v_r = 0 \end{equation}
\[ \begin{aligned}
\text{\ref{eq:2.2.10.3}} - \text{\ref{eq:2.2.10.2}}
& \implies \underbrace{(\lambda_r - \lambda_1)}{\neq0} \mu_1 v_1 + \cdots + \underbrace{(\lambda_r - \lambda_{r-1})}{\neq0} \mu_{r-1} v_{r-1} = 0 \\
& \implies v_1, \dots, v_{r-1} \text{ linear abhängig. \Lightning}
\end{aligned} \]
\end{itemize}
\end{proof}
\end{lemma}
\begin{lemma}
Sei $\alpha \in \homk, \dim(V)=n \text{ oder } A \in \K^{\nxn}$ mit $n$ verschiedenen Eigenvektoren. Dann ist $\alpha/A$ diagonalisierbar.
\begin{proof}
Wegen Lemma \ref{theo:2.2.10} gibt es Basis von Eigenvektoren. Daher ist $\alpha/A$ diagonalisierbar wegen Lemma \ref{theo:2.2.2}.
\end{proof}
\end{lemma}
\begin{defin}
Sei $\spec(A) = \{\lambda_1, \dots, \lambda_r \}$ und $(\lambda_1 - \lambda)^{k_1} \cdots (\lambda_r - \lambda)^{k_r} p \in\K[X]$ mit $p$ nicht durch Linearfaktoren teilbar (also keine Nullstellen in $\K$).\\
$k_i$ heißt \underline{algebraische Vielfachheit} des Eigenwerts $\lambda_i$. Wir schreiben $k_i = m_a(\lambda_i)$.\\
$\dim(\eig_A(\lambda_i))$ heißt \underline{geometrische Vielfachheit} des Eigenwerts $\lambda_i$. Wir schreiben $\dim(\eig_A(\lambda_i)) = m_g(\lambda_i)$
\end{defin}
\subsubsection{Beispiel}
\begin{itemize}
\item $\chi_A(\lambda) = \lambda^4 - 2 \lambda^3 + 2 \lambda^2 - 2\lambda + 1 \in \mathbb{R}[X]$\\
$\implies \chi_A(\lambda) = (1 - X)^2 \underbrace{(1 + \lambda^2)}{p(\lambda)}$ \\
$\implies m_a(1) = 2$
\item Für $\K=\mathbb{C}$ zerfällt jedes Polynom in Linearfaktoren, also ist $p$ immer konstant.
\end{itemize}
\begin{satz}
Sei $\mu\in\spec(A)/\spec(\alpha)$. Dann gilt \[ 1\le m_g(\mu) \le m_a(\mu) \]
\begin{proof}
Klarerweise gilt $1\le m_g(\mu)$ da $\mu$ Eigenwert ist. Sei $r:= m_g(\mu)$ und $b_1, \dots, b_r$ Basis von $\eig_\alpha(\mu)$. Sei $B=(b_1, \dots, b_n)$ Basis. Dann ist\\ ${}_B M(\alpha)_B =
\bordermatrix{
& & & & r & & \cr
& \mu & 0 & 0 & 0 & * & \dots & * \cr
& 0 & \mu & 0 & 0 & * & \dots & * \cr
& \vdots & & \ddots & \vdots & \vdots & & \vdots \cr
r & 0 & 0 & 0 & \mu & * & \dots & * \cr
& \vdots & \vdots & \vdots & \vdots & \vdots & & \vdots \cr
& 0 & 0 & 0 & 0 & * & \dots & *
}
$, also \[\begin{aligned}
\chi_\alpha(\lambda) & = \left\lvert \begin{array}{c | c}
\begin{smallmatrix}\mu - \lambda & & \\ & \ddots & \\ & & \mu - \lambda\end{smallmatrix} & A \\
\hline \\
0 & B
\end{array} \right\rvert \underbrace{=}_{\text{Satz \ref{theo:1.4.10}}} \det
\begin{pmatrix}
\mu - \lambda & & 0 \\
& \ddots & \\
0 & & \mu - \lambda
\end{pmatrix} \cdot \det(B) \\
& = (\mu - \lambda)^r \det(B) \\
& \implies r \le m_a(\mu)
\end{aligned}\]
\end{proof}
\end{satz}
\begin{lemma}
Seien $A, B$ ähnlich und $\mu \in \spec(A) (=\spec(B) \text{ nach Lemma \ref{theo:2.2.7}})$. Dann stimmen die geometrischen Vielfachheiten überein, das heißt $\dim(\eig_\mu(A)) = \dim(\eig_\mu(B))$.
\begin{proof}
Sei $B = P^{-1} A P$. Dann gilt \[ \begin{aligned}
\eig_{\mu}(B) & = \ker(B - \mu I) = \ker(B - \mu P^{-1} P) \\
& = \ker(P^{-1} (A - \mu I) P) \\
& \underbrace{\implies}_{\mathclap{\text{Für ähnliche Matrizen stimmen die Dimensionen der Kerne überein}}} \dim(\eig_\mu(B)) = \dim\eig_\mu(A)
\end{aligned}\]
\end{proof}
\end{lemma}
\begin{satz}
$A/\alpha$ diagonalisierbar $\iff$
\begin{enumerate}[label=\roman*)]
\item $\chi_{A/\alpha}$ zerfällt in Linearfaktoren, das heißt
\[\chi_{A/\alpha}(\lambda)= (\lambda_1 - \lambda)^{k_1} \cdots (\lambda_r - \lambda)^{k_r}, \sum k_i = n\]
\item algebraische und geometrische Vielfachheiten stimmen überein, das \\
heißt $m_a(\lambda_i) = m_g(\lambda_i), i=1, \dots, r$
\end{enumerate}
\begin{proof}
\begin{itemize}
\item[$\impliedby$:] Aus i), ii) folgt, dass \begin{equation}\label{eq:2.2.15.1}
\sum_{i=1}^r \underbrace{\dim(\eig_\alpha(\lambda_i))}_{=m_g(\lambda_i)=:d_i} = n \end{equation}
Sei $b_i^1, \dots, b_i^{d_i}$ Basis von $\eig_\alpha(\lambda_i)$. Wir zeigen, dass $B=\{b_i^1, \dots, b_i^{d_i}: i=1, \dots, r\}$ Basis ist.
\begin{enumerate}[label=\arabic*)]
\item $\lvert B \rvert = n$ folgt aus \ref{eq:2.2.15.1}
\item Ang. $\sum\limits_{i=1}^r (\underbrace{\mu_i^1 b_i^1 + \cdots + \mu_i^{d_i} b_i^{d_i}}_{v_i}) = 0$ \\
$\underbrace{\implies}_{\mathclap{\substack{v_i \text{Eigenwerte zu} \\ \text{verschiedenen Eigenvektoren} \\ + \text{Lemma \ref{theo:2.2.10}}}}}
v_i = 0 \forall i=1, \dots, r \underbrace{\implies}_{\mathclap{\substack{b_i^1, \dots, b_i^{d_i} \\ \text{Basis von } \eig_\alpha(\lambda_i)}}} \mu_i^1, \dots, \mu_i^{d_i} = 0 \forall i=1, \dots, r$ \\
$ \implies B $ ist Basis aus Eigenvektoren $\underbrace{\implies}_{\mathclap{\text{Lemma \ref{theo:2.2.2}}}} \alpha $ diagonalisierbar.
\end{enumerate}
\item[$\implies$:] Sei $\alpha$ diagonalisierbar. \[\begin{aligned}
& \implies \exists \text{ Basis } \{b_1, \dots, b_n\} \text{ aus Eigenvektoren} \\
& \implies {}_B M(\alpha)_B = \diag(\lambda_1, \dots, \lambda_n) \\
& \implies \chi_B(\lambda) = (\lambda_1 - \lambda) \cdots (\lambda_n - \lambda)
\end{aligned}\]
\end{itemize}
\end{proof}
\end{satz}
\subsubsection{Diagonalisieren}
\begin{enumerate}[label=\arabic*)]
\item Zerlegung in Linearfaktoren
\[ \chi_A(\lambda) = (\lambda_1 - \lambda)^{m_a(\lambda_1)} \cdots (\lambda_r - \lambda)^{m_a(\lambda_r)} \]
\item Bestimme Basis $B_i$ der Eigenräume
\[ \eig_A(\lambda_i) = \ker(A - \lambda_i I) \]
\item Ordne Basis $B= \bigcup\limits_{i=1}^n B_i$ zu $B= (b_1, \dots, b_n)$
\item Mit $S = (b_1, \dots, b_n)$ gilt dann \[
\diag(\underbrace{\lambda_1, \dots, \lambda_n}_{\mathclap{\substack{\text{Eigenwerte werden nach} \\ \text{Vielfachheit gezählt!} \\ \lambda_i \text{ ist Eigenwert von } b_i \text{!}}}}) = S^{-1} A S
\]
\end{enumerate}
\subsubsection{Beispiel}
$A = \begin{pmatrix}
1 & 2 & 2 \\
2 & -2 & 1 \\
2 & 1 & -2
\end{pmatrix}$
\begin{enumerate}[label=\arabic*)]
\item \[\begin{aligned}
\chi_A(\lambda) = & \begin{vmatrix}
1 -\lambda & 2 & 2 \\
2 & -2 -\lambda & 1 \\
2 & 1 & -2 -\lambda
\end{vmatrix} \\
\underbrace{=}_{\mathclap{\substack{\text{Entwicklung} \\ \text{nach 1. Zeile}}}}
& (1-\lambda) \begin{vmatrix} -2 -\lambda & 1 \\ 1 & -2 -\lambda \end{vmatrix}
+ (-2) \begin{vmatrix} 2 & 1 \\ 2 & -2-\lambda \end{vmatrix} \\
& + 2 \begin{vmatrix} 2 & -2 - \lambda \\ 2 & 1 \end{vmatrix} \\
= & \dots= -\lambda^3 - 3 \lambda^2 + 9\lambda + 27 = (3-\lambda)(-3-\lambda)^2
\end{aligned}\]
\item $\lambda = 3$
\[\begin{aligned}
& \left( \begin{array}{c c c | c} 1-3 & 2 & 2 & 0 \\ 2 & -2-3 & 1 & 0 \\ 2 & 1 & -2-3 & 0 \end{array} \right)
= \left( \begin{array}{c c c | c} -2 & 2 & 2 & 0 \\ 2 & -5 & 1 & 0 \\ 2 & 1 & -5 & 0 \end{array} \right) \\
& \sim \left( \begin{array}{c c c | c} -1 & 1 & 1 & 0 \\ 0 & -3 & 3 & 0 \\ 0 & 3 & -3 & 0 \end{array} \right)
\sim \left( \begin{array}{c c c | c} 1 & -1 & -1 & 0 \\ 0 & 1 & -1 & 0 \\ 0 & 0 & 0 & 0 \end{array} \right)
\sim \left( \begin{array}{c c c | c} 1 & 0 & -2 & 0 \\ 0 & 1 & -1 & 0 \\ 0 & 0 & 0 & 0 \end{array} \right) \\
& \implies \eig_A(3) = \left\langle \begin{pmatrix} 2 \\ 1 \\ 1 \end{pmatrix} \right\rangle
\end{aligned}\]
$\lambda = -3$
\[\begin{aligned}
& \left( \begin{array}{c c c | c} 1+3 & 2 & 2 & 0 \\ 2 & -2+3 & 1 & 0 \\ 2 & 1 & -2+3 & 0 \end{array} \right)
= \left( \begin{array}{c c c | c} 4 & 2 & 2 & 0 \\ 2 & 1 & 1 & 0 \\ 2 & 1 & 1 & 0 \end{array} \right) \\
& \sim \left( \begin{array}{c c c | c} 2 & 1 & 1 & 0 \\ 0 & 0 & 0 & 0 \\ 0 & 0 & 0 & 0 \end{array} \right) \\
& \implies \eig_A(-3) = \left\langle \begin{pmatrix} -1 \\ 0 \\ 2 \end{pmatrix}, \begin{pmatrix} -1 \\ 2 \\ 0 \end{pmatrix} \right\rangle
\end{aligned}\]
\item \[\begin{aligned}
& S = \begin{pmatrix} 2 & -1 & -1 \\ 1 & 0 & 2 \\ 1 & 2 & 0 \end{pmatrix} \\
& \implies S^{-1} A S = \begin{pmatrix} 3 & 0 & 0 \\ 0 & -3 & 0 \\ 0 & 0 & -3 \end{pmatrix}
\end{aligned}\]
\end{enumerate}
\begin{lemma} \label{theo:2.2.16}
Sei $A\in\K^{\nxn}$ und $\underbrace{\spur(A)}_{\mathclap{\color{red}\text{\dq Spur von $A$ \dq}}} := \sum\limits_{i=1}^n a_{ii}$
\[\chi_A(\lambda) = (-1)^n\lambda^n + (-1)^{n-1} \spur(A) \lambda^{n-1} + \cdots + \det(A)\]
\begin{proof}
$\chi_A(\lambda) = \sum\limits_{\pi \in S_n} \sgn(\pi) \prod\limits_{i=1}^n \tilde{a}_{i\pi(i)}$ mit $\tilde{a}_{ij} = \begin{cases} a_{ij} & i\neq j \\ a_{ij} - \lambda & i=j\end{cases}$. \\
Wenn $\pi\neq \id$ gilt $\deg\left(\prod\limits_{i=1}^n \tilde{a}_{i\pi(i)}\right)\le n-2$, da mindestens zwei Elemente vertauscht werden. Die Koeffizienten von Grad $n, n-1$ kann man also aus $\prod\limits_{i=1}^n \tilde{a}_{ii} = \prod\limits_{i=1}^n (\tilde{a}_{ii} - \lambda)$ ablesen. Daraus folgt die Behauptung für die höchsten beiden Koeffizienten. Weiters gilt $\chi_A(0)=\det(A)$, was die Aussage für den konstanten Koeffizienten zeigt.
\end{proof}
\end{lemma}
$\sigma_j := (-1)^j \sum\limits_{\substack{S\subset [n] \\ \lvert S \rvert = n-j}} \prod\limits_{s \in S} \lambda_s$
\begin{korollar}
\leavevmode
\begin{enumerate}[label=\alph*)]
\item $A\sim B \implies \spur(A)=\spur(B)$
\item A diagonalisierbar $\implies \spur(A)=\lambda_1 + \cdots + \lambda_n$ mit $\lambda_1, \dots, \lambda_n$ Eigenwerte von $A$, nach Vielfachheit gezählt.
\item A diagonalisierbar $\implies \det(A)=\lambda_1 \cdot \dots \cdot \lambda_n$ mit $\lambda_1, \dots, \lambda_n$ Eigenwerte von $A$, nach Vielfachheit gezählt.
\end{enumerate}
\begin{proof}
Folgt daraus, dass das charakteristische Polynom (und damit seine Koeffizienten) unter Ähnlichkeit invariant sind (Lemma \ref{theo:2.2.7}) und Lemma \ref{theo:2.2.16}
\end{proof}
\end{korollar}
\begin{satz}[Cayley-Hamilton]
\dq$\chi_A(A) = 0$\dq, das heißt sei $A\in \K^{\nxn}$ mit charakteristischem Polynom $\chi_A(\lambda)=c_n \lambda^n + c_{n-1} \lambda^{n-1} + \cdots + c_0$.
Dann gilt
\[
\chi_A(A):=c_n A^n + c_{n-1} A ^{n-1} + \cdots c_0 I = 0 = \begin{pmatrix}0 &\dots &0 \\ \vdots& \ddots &\vdots \\ 0 & \dots & 0\end{pmatrix} \in \K^{\nxn}
\]
\begin{proof}
Sei $B := A^T - \lambda I =
\begin{pmatrix}
a_{11} - \lambda & a_{21} & \dots & a_{n1} \\
a_{12} & a_{22} - \lambda & \dots & a_{n2} \\
\vdots & \ddots & \ddots & \vdots \\
a_{1n} & a_{2n} & \dots & a_{nn} - \lambda
\end{pmatrix}
= (a_{ji} - \delta_{ij} \lambda)_{ij}$
und $C:= \adj(B)$, sodass
\begin{equation}
CB = \det(B) I_n = \chi_A = I_n [\chi_A = \chi_{A^T}
\label{eq:2.2.18.1}
\end{equation}
\ref{eq:2.2.18.1} heißt komponentenweise, dass
\begin{flalign}
& \sum_{i=1}^{n}
\underbrace{c_{ki}}_{\mathrlap{\text{Polynome, in die $A$ eingesetzt werden kann}}}
\underbrace{b_{ij}}
= \delta_{ij} \cdot \underbrace{\chi_A} \forall k, j \in [n] \nonumber \\
= & \sum_{i=1}^{n}c_{ki}(A) b_{ij}(A) = \delta_{jk}\chi_A (A) \label{eq:2.2.18.2}
\end{flalign}
Wegen $b_{ij}(A) = a_{ji} I_N - \delta_{ij}A$ gilt weiters
\begin{equation}
\forall i \in [n]: \sum_{j=1}^{n} b_{ij}(A) e_j = (\sum_{j=1}^{n} a_{ji} e_j) - A e_i = 0
\label{eq:2.2.18.3}
\end{equation}
Es folgt $\forall k \in [n]$
\begin{flalign*}
\chi_A (A) e_k & = \sum_{j=1}^{n} \delta_{jk} \chi(A) e_j & \\
& \underbrace{=}_{\mathclap{\text{\ref{eq:2.2.18.2}}}}
\sum_{j=1}^{n} \sum_{i=1}^{n} c_{ki}(A) b_{ij}(A) e_j & \\
& = \sum_{i=1}^{n} c_{ki}(A) (\sum_{j=1}^{n} b_{ij(A) e_j}) & \\
& \underbrace{=}_{\mathclap{\text{\ref{eq:2.2.18.3}}}} 0 & \\
\implies \chi_A(A) = 0
\end{flalign*}
\end{proof}
\end{satz}
\subsubsection{Berechnung der Koeffizienten von $\chi_A$}
Sei $f(\lambda) \underbrace{=}_{\text{(*)}} \prod\limits_{j=1}^{n}(\lambda_j - \lambda) = \underbrace{c_n\lambda^n}_{=(-1)^n} + c_{n-1}\lambda ^{n-1} + \cdots + c_0$
Wie können wir $c_j$ effizient bestimmen?
\begin{itemize}
\item [Bemerkung 1:] $\displaystyle { c_j = (-1)^{j} \sum_{\substack{S\subseteq [n] \\
\lvert S \rvert = n-j}} \prod_{s \in S} \lambda_s =:
\sigma_{n-j}^n (\lambda_1, \dots, \lambda_n)}$ \\
Dies folgt aus (*) durch Ausmultiplizieren \\
Sei nun weiters $p_j^n(\lambda_1, \dots, \lambda_n) := \sum\limits_{i=1}^{n}\lambda_i^j$
\item [Bemerkung 2:] $\sigma_j^n, p_j^n$ sind symmetrisch, das heißt
\[\begin{aligned}
& \sigma_j^n(\lambda_{\pi(1)}, \dots, \lambda_{\pi(n)}) = \sigma_{j}^n (\lambda_1, \dots, \lambda_n) \\
& p_j^n(\lambda_{\pi(1)}, \dots, \lambda_{\pi(n)}) = p_{j}^n (\lambda_1, \dots, \lambda_n)
\end{aligned} \text{ für } \pi \in S_n\]
\end{itemize}
\begin{lemma}[Newtonidentität] \label{theo:2.2.19}
Es gilt für $k\le n$
\[k\sigma_k^n+\sum_{j=0}^{k-1}\sigma_j^n p_{k-j}^n=0\]
\begin{proof}
Induktion.
\begin{itemize}
\item [$k=n$:] Wegen
\begin{equation*}
0= \sum_{i=1}^{n} =
\sum_{i=1}^{n} \sum_{j=0}^n c_j \lambda_i^j =
\sum_{j=0}^n c_j p_j^n =
\sum_{j=0}^n \sigma_{n-j}^n p_j^n =
\sum_{j=0}^n \sigma_j^n p_{n-j}^n
\end{equation*}
folgt $\sigma_n^n p_0^n + \sum\limits_{j=0}^n \sigma_j^n p_{n-j}^n = 0$ was mit
$p_0^n = n$ die gewünschte Aussage liefert.
\item [$k<n$:] Betrachte das (symmetrische) \[
q(\lambda_1, \dots, \lambda_n) :=
k \sigma_k^n + \sum_{j=0}^{k-1} \sigma_j^n p_{k-1}^n
\]
Es gilt \[q(\lambda_1, \dots, \lambda_n) =
\sum_{j_1, \dots, j_n} c_{j_1 \dots j_n} \lambda_1^{j_1} \cdots \lambda_n^{j_n}\]
und wir müssen zeigen, dass alle Koeffizienten $c_{j_1 \dots j_n}=0$ sind.
Dazu bemerken wir, dass $c_{j_1 \dots j_n}$ immer $0$ ist,
wenn mehr als $k$ $j_i$'s ungleich $0$ sind.\\
Sei also $c_{j_1 \dots j_n}$ ein solcher Koeffizient mit $j_{k+1}, \dots, j_n=0$.
Dann gilt
\begin{align*}
& \underset{\rotatebox{90}{$=$}}
{q(\lambda_1, \dots, \lambda_k, 0, \dots, 0)} =
\sum_{j_1, \dots, j_k} c_{j_1 \dots j_n 0 \dots 0}
\lambda_1^{j_1} \cdots \lambda_n^{j_k} \\
& k \sigma_k^n + \sum_{j=0}^{k-1} \sigma_j^k p_{k-1}^k = 0
\text{ nach Voraussetzung}
\end{align*}
Aufgrund der Symmetrie gilt dasselbe Argument für alle anderen Koeffizienten
mit höchstens $k$ vielen $j_i$'s ungleich $0$.
\end{itemize}
\end{proof}
\end{lemma}
\begin{satz}
Sei $A \in \K^{\nxn}$ diagonalisierbar. Dann gilt für
\begin{align*}
\chi_A(\lambda) & = c_{n}\lambda^{n} + c_{n-1} \lambda ^{n-1} + \cdots + c_0 \\
& c_n = (-1)^n \\
& c_{n-k} = -\frac1k \sum_{j=0}^{k-1} c_{n-j} \spur(A^{k-j})
\end{align*}
\begin{proof}
Folgt direkt aus Lemma \ref{theo:2.2.19} und der Bemerkung dass für $A$ diagonalisierbar \\
$\spur(A^k) = \lambda_1^k + \cdots + \lambda_n^k$ gilt.
\end{proof}
\end{satz}
\subsubsection{Bemerkung}
$\underset{\mathrlap{\text{\dq fast alle Matrizen sind diagonalisierbar\dq}}}
{\text{Gilt}}$ auch für $A$ nicht diagonalisierbar. \dq Beweis\dq Stetigkeit
\subsubsection{Triangulierbarkeit von Matrizen}
\begin{defin}
\leavevmode
\begin{enumerate}[label=\alph*)]
\item $\alpha \in \homk, \dim(V)=n$ heißt \underline{triangulierbar} wenn es eine Basis $B$ gibt,
sodass ${}_B M(\alpha)_B$ obere Dreiecksgestalt hat.
\item $A\in\K^{\nxn}$ heißt \underline{triangulierbar} wenn es eine reguläre Matrix $P\in\K^{\nxn}$ gibt,
mit $P^{-1} A P$ obere Dreiecksgestalt.
\end{enumerate}
\end{defin}
\begin{satz}
$A \in \K^{\nxn} / \alpha$ ist triangulierbar $\iff \chi_A / \chi_\alpha$ zerfällt in Linearfaktoren.
\begin{proof}[Beweis]
\begin{itemize}
\item[$\implies$:] $\chi_A$ ist invariant
bezüglich Ähnlichkeitsumformung (Lemma \ref{theo:2.2.7}).
Sei $P^{-1} A P = \begin{pmatrix} \lambda_1 & & * \\
& \ddots & \\
0 & & \lambda_n\end{pmatrix}$
, dann folgt\\
$\chi_A(\lambda) = \chi_{P^{-1} A P}(\lambda)
= \prod\limits_{i=1}^n (\lambda_i - \lambda) $
\item[$\impliedby$:] Induktion nach $n$
\begin{itemize}
\item[$n=1$:] Jede $1\times1$ Matrix ist obere Dreiecksmatrix.
\item[$n-1\mapsto n$:] Sei $\chi_A(\lambda) =
\prod\limits_{i=1}^n (\lambda_i - \lambda)$ und sei
$b_1 \in \eig_{\lambda_1}(\alpha)$.
Sei $B=(b_1, \dots, b_n)$ Basis von $\K^n$. Dann gilt
\[\begin{aligned}
& A
= {}_B M(\alpha)_B
= \begin{pmatrix}
\lambda_1 & a_{12} & \dots & a_{1n} \\
0 & & & \\
\vdots & & \tilde{A} & \\
0 & & &\end{pmatrix} \\
& \text{Sei }\beta: \begin{cases}
\overbrace{\langle b_2, \dots, b_n\rangle}^{V}
& \to \langle b_2, \dots, b_n\rangle \\
b_i
& \mapsto \Phi^{-1}_{\tilde{B}}(C\cdot
{}_{\tilde{B}}v)
\end{cases}
\end{aligned}\]
Es gilt $\chi_\alpha(\lambda) =
\lambda_1 - \lambda) \cdot \chi_\beta(\lambda)$,
daher zerfällt $\chi_\beta$ in Linearfaktoren.
Nach Induktionsvoraussetzung existiert eine Basis $\tilde{B} =
(\tilde{b}_2, \dots, \tilde{b}_n)$ von $\tilde{V}$ mit
\begin{equation}
{}_{\tilde{B}} M(\beta)_{\tilde{B}} =
\begin{pmatrix} \lambda_2 & & * \\
& \ddots & \\
0 & & \lambda_n\end{pmatrix}
\label{eq:2.2.22.1}
\end{equation}
Weiters ist $\alpha(b_i) = a_{1i} b_1 + \beta(b_i), i=2, \dots, n$.
Sei $\tilde{b}_i = \sum\limits_{j=2}^n \mu_{ij} b_j$.
Wegen \ref{eq:2.2.22.1} gilt
\begin{equation}
\beta(\tilde{b}_i) \in
\langle \tilde{b}_1, \dots, \tilde{b}_i \rangle
\label{eq:2.2.22.2}
\end{equation}
Wir zeigen nun, dass für die Basis $C=(c_1, \dots, c_n)$ mit
$c_1 = b_1, c_2 = \tilde{b}_2, \dots, c_n = \tilde{b}_i $
die Matrix ${}_C M(\alpha)_C$ obere Dreiecksgestalt hat.
Dies ist äquivalent zu
\[\alpha(c_i)\in \langle c_1, \dots, c_n \rangle \forall i=1, \dots, n \]
\begin{itemize}
\item [$i=1$:] $\alpha(c_1) = \alpha(b_1) = \lambda_1 b_1
\in \langle b_1 \rangle = \langle c_1 \rangle$
\item [$i>1$:]
\begin{align*}
& \alpha(c_i) = \alpha(\tilde{b}_i) =
\alpha(\sum_{j=2}^n \mu_{ij} b_j)
= \sum_{j=2}^n \mu_{ij} \alpha(b_j) \\
& = \sum_{j=2}^n\mu_{ij}(a_{1j} b_1 + \beta(b_j))
= (\underbrace{\sum_{j=2}^n \mu_{ij} a_{1j}}_
{\displaystyle\sigma_i})
+ \sum_{j=2}^n \mu_{ij}\beta(b_j) \\
& = \sigma_i b_1+ \beta(\sum_{j=2}^n \mu_{ij} b_j)
= \sigma_i b_1 + \beta(\tilde{b}_i) \\
& \underbrace{\in}_{\text{\ref{eq:2.2.22.2}}}
\langle b_1,\tilde{b}_2,\dots,\tilde{b}_i\rangle
= \langle c_1, \dots, c_i \rangle
\end{align*}
\end{itemize}
\end{itemize}
\end{itemize}
\end{proof}
\end{satz}
\section{Jordan Normalform}
\begin{defin}
Eine $m\times m$ Matrix
\[J_m(\lambda) := \begin{pmatrix}
\lambda & 1 & 0 & \dots & 0 \\
0 & \ddots & \ddots & \ddots & \vdots \\
\vdots & \ddots & \ddots & \ddots & 0 \\
\vdots & \ddots & \ddots & \ddots & 1 \\
0 & \dots & \dots & 0 & \lambda
\end{pmatrix}\]
heißt \underline{Jordanblock} der Dimension $m$ zum Eigenwert $\lambda$.\\
Eine Matrix $A \in \K^{\nxn}$, die als Blockdiagonalmatrix aus Jordanblöcken besteht,
heißt \underline{Jordanmatrix}. \\
$A \in \K^{\nxn}$ besitzt eine \underline{Jordan-Normalform} wenn $P\in\K^{\nxn}$ invertierbar existiert,
sodass $P^{-1}AP$ Jordanmatrix ist.\\
$\alpha \in \homk$ besitzt eine \underline{Jordan-Normalform} wenn eine Basis $B$ von $V$ existiert,
sodass $ {}_{B} M(\alpha)_{B} $ Jordanmatrix ist.\\
B heißt \underline{Jordanbasis} zu $A/\alpha$.
\end{defin}
\subsubsection{Beispiel}
\begin{itemize}
\item Jede Diagonalmatrix ist Jordanmatrix
\item $\begin{pmatrix}1\end{pmatrix},
\begin{pmatrix}1 & 1 \\ 0 & 1\end{pmatrix},
\begin{pmatrix}0 & 1 \\ 0 & 0\end{pmatrix},
\begin{pmatrix}1 & 1 & 0 \\ 0 & 1 & 0 \\ 0 & 0 & 2\end{pmatrix},
\xcancel{\begin{pmatrix} 1 & 1 \\ 0 & 2 \end{pmatrix}}
$
\end{itemize}
Wir wollen zeigen, dass $\alpha/A$ genau dann eine Jordan-Normalform besitzt, wenn $\alpha/A$ triangulierbar ist.
\subsubsection{Bemerkung}
\begin{itemize}
\item $\chi_{J_m(\lambda)}(\mu) = (\lambda - \mu)^m \implies \spec(J_m(\lambda)) = \{\lambda\}$ \\
$J_m(\lambda) - \lambda I = \begin{pmatrix}
0 & 1 & 0 & \dots & 0 \\
0 & \ddots & \ddots & \ddots & \vdots \\
\vdots & \ddots & \ddots & \ddots & \vdots \\
\vdots & & \ddots & \ddots & 1 \\
0 & \dots & \dots & 0 & 0
\end{pmatrix}$\\
$\implies \dim(\eig_{J_m(\lambda)}(\lambda)) = \dim(\ker(J_m(\lambda) - \lambda I)) = 1$ \\
$\implies m_g(\lambda) = 1$ und $m_a(\lambda) = m$.
\item $J_m(0)^m = 0$, das heißt $J_m(0)$ ist \underline{nilpotent}.
\begin{align*}
& J_m(0)(e_i): \begin{cases} e_{i-1} & i \in \{2, \dots, m\} \\
0 & \text{sonst}\end{cases} \\
& J_m(0)^l(e_i): \begin{cases}e_{i-l} & i \in \{l+1, \dots, m\} \\
0 & \text{sonst}\end{cases}
\end{align*}
\end{itemize}
\begin{defin}
$\alpha \in \homk$ oder $A\in \K^{\nxn}$ heißt \underline{nilpotent} (mit Index $m$) falls
$\alpha^m = 0 / A^m = 0$ und $\forall l \in [m-1]: \alpha^l \neq 0 / A^l \neq 0$.
\end{defin}
\begin{lemma}
\label{theo:2.3.3}
Sei $\alpha \in \homk, \dim(V)=n$ nilpotent mit Index $m$. Dann existiert eine Basis $B$ mit
\begin{equation*}
{}_B M(\alpha)_B =
\begin{pmatrix}
0 & \delta_1 & & \\
& \ddots & \ddots & \\
& & \ddots & \delta_{n-1} \\
& & & 0
\end{pmatrix}
\text{ und } \delta_i \in \{0, 1\} \forall i \in [n-1]
\end{equation*}
Das heißt ${}_B M(\alpha)_B$ ist blockdiagonal mit Jordanblöcken mit Eigenwerten $0$
\begin{proof}
Sei $V_i := \ker(\alpha^i)$. \\
Dies ergibt eine aufsteigende Kette von Unterräumen
\begin{equation*}
\underbrace{\{0\}}_{=V_0} \subseteq V_1 \subseteq \cdots \subseteq \underbrace{V_m}_{=V}
\end{equation*}
Wir bauen uns iterativ eine Basis für Komplemente $W_i$ mit $V_{i-1} \oplus W_i = V_i$.
Sei also $B^{m-1}$ Basis von $V_{m-1}$. \\
$C^m = (c_1^m, \dots, c_{r_{m}})$ Basis von $W_m$
[das heißt $C^m$ ergänzt die Basis $B^{m-1}$ zu Basis von $V^m$]. \\
\underline{Behauptung}
\begin{enumerate} [label=\arabic*)]
\item $\alpha(C^m) \subseteq V_{m-1}$
\item $\alpha(C^m)$ linear unabhängig
\item $\langle \alpha(C^m) \rangle \cap V_{m-2} = \{0\}$
\end{enumerate}
\begin{proof}[Zwischenbeweis]
\begin{itemize}
\item[1)] folgt aus $\alpha(V_{i+1}) \subseteq \alpha(V_i)$
\item[3)] Sei $\sum\limits_{i}\mu_i \alpha(c_i^m) \in V_{m-2}$
\begin{align*}
& \implies \alpha^{m-2}(\sum_{i}\mu_i \alpha(c_i^m)) = 0 \\
& \implies \alpha^{m-1} (\sum_{i} \mu_i \alpha(c_i^m)) = 0 \\
& \implies \sum \mu_i c_i^m \in V_{m-1} \\
& \underbrace{\implies}_{\mathclap{\substack{(c_i^m) \text{ liegen} \\
\text{im Komplement} \\
\text{von } V_{m-1}}}}
\mu_i = 0, \forall i \implies \sum_{i} \mu_i \alpha(c_i^m) = 0
\end{align*}
\item[2)] folgt aus 3) [da $0\in V_{m-2}$]
\end{itemize}
\end{proof}
Es folgt, dass \begin{align*}
& & \langle D^{m-1} \rangle\;\; & \langle D^m \rangle \\
& & \underbrace{V_{m-2} \oplus \langle \alpha(C^m) \rangle \oplus \langle C^{m-1} \rangle}_{V_{m-1}}
\oplus & \langle C^m \rangle = V
\end{align*}
Setze $D^m := C^m$ und definiere induktiv für $D^i \subseteq V_i$ die Menge
$D^{i-1} := \alpha(D^i) \cup C^{i-1} \subseteq V_{i-1}$ sodass mit einer Basis $B^{i-2}$ von
$V_{i-2}$ die Menge $B^{i-2} \cup D^{i-1}$ Basis von $V_{i-1}$ ist, also
\[
V_{i-2} \oplus \underbrace{\langle \alpha(D^i) \rangle \oplus \langle C^{i-1} \rangle}_{\langle D^i \rangle}
= V_{i-1} \text{$\leftarrow$ das geht nach obiger Behauptung}
\]
Nach Konstruktion ist $(D^1, \dots, D^m)$ Basis von $V$.
Sie besteht aus folgenden Elementen:
\begin{align*}
\left.
\begin{array}{lll}
J_m(0) \to & \alpha^{m-1}(d_1^m), \dots, \alpha(d_1^m), & d_1^m \\
& & \vdots \\
J_m(0) \to & \alpha^{m-1}(d_{r_m}^m), \dots, \alpha(d_{r_m}^m), & d_{r_m}^m
\end{array}
\right\} \in V_m \\
\left.
\begin{array}{lll}
J_{m-1}(0) \to & \alpha^{m-1}(d_1^{m-1}), \dots, \alpha(d_1^{m-1}), & d_1^{m-1} \\
& & \vdots \\
J_{m-1}(0) \to & \alpha^{m-1}(d_{r_{m-1}}^{m-1}), \dots, \alpha(d_{r_{m-1}}^{m-1}), & d_{r_{m-1}}^{m-1}
\end{array}
\right\} \in V_{m-1} \\
\left. \begin{array}{lr}
J_1(0) \to & d_1^1 \\
& \vdots \\
J_1(0) \to & d_{r_1}^1
\end{array}
\right\} V_1 = \ker(\alpha)
\end{align*}
Wenn wir die Basiselemente von links nach rechts und von oben nach unten ordnen erhalten wir
die gewünschte Gestalt.
\end{proof}
\end{lemma}
\subsubsection{Bemerkung}
Angenommen \(\alpha - \lambda \id: V \to V\) nilpotent. Dann besitzt \(\alpha\) nach Lemma
\ref{theo:2.3.3} Jordan-Normalform.
\begin{defin}
\label{theo:2.3.4}
Sei \(V \K\)-Vektorraum, \(\dim(V) < \infty, \alpha \in \homk\) und \(\lambda \in \spec(\alpha)\).
Für \(l \in \mathbb{N}\) definiere \(V_{l, \lambda}:= \ker((\alpha - \lambda \id)^l)\)
\end{defin}
\subsubsection{Bemerkung}
\begin{itemize}
\item $\alpha - \lambda \id|_{V_{l, \lambda}} \in \homkk(V_{l, \lambda}, V_{l, \lambda})$:
\begin{align*}
\text{zu Zeigen: } v\in V_{l, \lambda} & \implies \alpha(v) - \lambda v \in V_{l, \lambda}\text{, das heißt} \\
(\alpha - \lambda \id)^l v = 0 & \implies (\alpha - \lambda \id)^{l-1} (\alpha - \lambda \id) v = 0 \\
& \implies (\alpha - \lambda \id)(v) \in V_{l, \lambda} & \square
\end{align*}
\item Nach Lemma \ref{theo:2.3.3} gibt es also Basis von $V_{l, \lambda}$ bezüglich derer
$\alpha - \lambda \id |_{V_{l, \lambda}}: V_{l, \lambda} \to V_{l, \lambda}$
Jordan-Normalform hat
\end{itemize}
\begin{lemma}
\label{theo:2.3.5}
Sei $V \K$-Vektorraum, $\dim(V) < \infty, \alpha \in \homk$. Für $l\in\mathbb{N}$ sei
$V_l := \ker(\alpha^l)$. Dann gilt $\alpha(V_l) \subseteq V_{l-1} \subseteq V_l$ für alle
$l\in \mathbb{N}$ und es existiert genau ein $k\in \mathbb{N}_0$ mit
\[
\{0\} = V_0 \subseteq V_1 \subseteq \cdots \subseteq V_k = V_{k+1} \text{ und } V_{l+1} = V_l, \forall l \ge k
\]
\begin{proof}
Da $\dim(V) < \infty$ muss es ein kleinstes $k$ mit $V_{k+1} = V_{k}$ geben.
Angenommen $\exists l\ge k$ mit $V_{l+1} \neq V_l$. Sei $0\neq v\in V_{l+1} \setminus V_l$
$\implies 0 = \alpha^{l+1}(v) = \alpha^{k+1}(\alpha^{l-k}(v))$ und $0\neq \alpha^l(v)
= \alpha^k (\alpha^{l-k}(v)) \implies 0\neq \alpha^{l-k}(v) \in V_{k+1}\setminus V_k$
\Lightning
\end{proof}
\end{lemma}
\begin{defin}
Sei $V_{l, \lambda}$ wie in Definition \ref{theo:2.3.4} und $k$ wie in Lemma \ref{theo:2.3.5}
Dann heißt \[
\widetilde{\eig_\alpha(\lambda)} := V_{k, \lambda} = V_{k+1, \lambda}
\]
\underline{verallgemeinerter Eigenraum} oder \underline{Hauptraum} von $\alpha$ zum Eigenwert
$\lambda$. $v \in V_{l, \lambda} \setminus V_{l-1, \lambda}$ für $1 \le l \le k$ heißt
\underline{verallgemeinerter Eigenvektor} der Ordnung $l$.
\end{defin}
\subsubsection{Idee}
\begin{itemize}
\item $\alpha|_{\widetilde{\eig_\alpha(\lambda)}}: \widetilde{\eig_\alpha(\lambda)} \to \widetilde{\eig_\alpha(\lambda)}$
hat Jordan-Normalform.
Zerlege \begin{equation}
\label{eq:2.3.6.1}
V:= \widetilde{\eig_\alpha(\lambda_1)} \oplus \cdots \oplus \widetilde{\eig_\alpha(\lambda_r)}
\end{equation}
dann besitzt ganz $\alpha: V\to V$ Jordan-Normalform
\item Sei $V= V_1 \oplus \cdots \oplus V_r$ und $\alpha \in \homk$. Falls $\alpha(V_i) \subseteq V_i$ für alle
$i \in [r]$, dann schreiben wir $\alpha = \alpha_1 \oplus \cdots \oplus \alpha_r$ mit $\alpha_i = \alpha|_{V_i} \forall i \in [r]$.
Für $v= v_1 + \cdots + v_r, v_i \in V_i, \forall i \in [r]$ gilt also $\alpha(v) = \alpha_1(v_1) + \cdots + \alpha_r(v_r)$.
Sei $B_i = \{b_1^i, \dots, b_{d_i}^i\}$ Basis von $V_i$ und $B = (B_1, \dots, B_r)$. Dann hat ${}_B M(\alpha)_B$
Blockdiagonalgestalt mit Blöcken ${}_{B_i} M(\alpha_i)_{B_i}$, das heißt \[
{}_B M(\alpha)_B = \begin{pmatrix}
\overbrace{{}_{B_1} M(\alpha_1)_{B_1}}^{\in \K^{d_1 \times d_1}} & & 0 \\
& \ddots & \\
0 & & \underbrace{{}_{B_r} M(\alpha_r)_{B_r}}_{\in\K^{d_r \times d_r}}
\end{pmatrix}
\]
Insbesondere gilt $\chi_\alpha = \chi_{\alpha_1} \cdot \dots \cdots \chi_{\alpha_r}$
\item Da wir schon wissen, dass $\alpha|_{\widetilde{\eig_\alpha(\lambda_i)}}$ Jordan-Normalform hat
folgt \\Jordan-Normalform für $\alpha$ wenn \ref{eq:2.3.6.1} gezeigt werden kann.
\end{itemize}
\begin{satz}
\label{theo:2.3.7}
Sei $V \K$-Vektorraum mit $\dim(V) < \infty$ und $\alpha \in \homk$ sodass
$\chi_\alpha(\lambda) = (\lambda_1 - \lambda) \cdots (\lambda_r - \lambda)$ in Linearfaktoren
zerfällt. Dann gilt
$V = \widetilde{\eig_\alpha(\lambda_1)} \oplus \cdots \oplus \widetilde{\eig_\alpha(\lambda_r)}$
und insbesondere $\alpha = \alpha \oplus \cdots \oplus \alpha_r$ mit
$\alpha_i := \alpha|_{\widetilde{\eig_\alpha(\lambda_i)}} \in \homkk(\widetilde{\eig_\alpha(\lambda_i)}, \widetilde{\eig_\alpha(\lambda_i)})$
\begin{proof}
Induktion nach $\dim(V)$.
\begin{itemize}
\item[$n=1$] \checkmark
\item[$n-1 \mapsto n$] Da $\chi_A$ in Linearfaktoren zerfällt besitzt es eine Nullstelle $\lambda \in \spec(\alpha)$.
\begin{enumerate}[label=Fall \arabic*:]
\item $\widetilde{\eig_\alpha(\lambda)} = V$ \\
\underline{Behauptung:} $\spec(\alpha) = \{\lambda\}$
\begin{proof}[Zwischenbeweis]
Angenommen $\lambda' \neq \lambda$ und $\lambda' \in \spec(\alpha)$ und $v\in \eig_\alpha(\lambda')$. \\
$\implies (\alpha - \lambda \id) (v) = \alpha(v) - \lambda'v + (\lambda' - \lambda) v = (\lambda' - \lambda)(v)$ \\
$\implies (\alpha - \lambda \id)^l (v) \neq 0,\forall l \in \mathbb{N}$\Lightning \\
Daraus folgt das gewünschte Resultat
\end{proof}
\item $\widetilde{\eig_\alpha(\lambda)} \neq V$. Sei $k$ minimal mit
$\ker(\alpha - \lambda - \id)^k = \widetilde{\eig_\alpha(\lambda)}$ [Lemma \ref{theo:2.3.5}]
Setze $V_1 := \widetilde{\eig_\alpha(\lambda)}, V_2 := \im(\alpha - \lambda \id)^k$. \\
\underline{Behauptung:}
\begin{enumerate}[label=(\roman*)]
\item $\alpha(V_i) \subseteq V_i, i \in \{1, 2\}$
\item $V = V_1 \oplus V_2$
\end{enumerate}
\begin{proof}[Zwischenbeweis]
\begin{enumerate}[label=(\roman*)]
\item Wir zeigen $(\alpha - \lambda \id)(V_i) \subseteq V_i$.
\begin{itemize}
\item[$i=1$:] Sei $v\in V_1 = \ker(\alpha - \lambda \id)^k$. Dann gilt klarerweise
$(\alpha - \lambda \id)(v) \in \ker(\alpha - \lambda \id)^k \checkmark$
\item[$i=2$:] Sei $v \in \im(\alpha - \lambda \id)^k$, also $v = (\alpha - \lambda \id)^k (w)$
$\implies (\alpha - \lambda \id)(v) = (\alpha - \lambda \id)^k (\alpha - \lambda \id)(w) \in \im(\alpha - \lambda \id)^k \checkmark$
\end{itemize}
\item Es gilt $\dim(V) = \dim(V_1) + \dim(V_2)$ nach der Dimensionsformel. Es genügt also zu zeigen, dass
$V_1 \cap V_2 = \{0\}$. Sei $v\in V_1 \cap V_2$
\begin{align*}
& \underbrace{\implies}_{v\in V_2} \exists w\in V: v = (\alpha - \lambda \id)^k(w) \\
& \underbrace{\implies}_{v\in V_1} (\alpha - \lambda \id)^{2k}(w) = 0 \\
& \implies w \in V_{2k, \lambda} \setminus V_{k, \lambda} \underbrace{\implies}_{\mathclap{\text{Lemma \ref{theo:2.3.5}}}} w = \{0\} \checkmark
\end{align*}
\end{enumerate}
\end{proof}
\end{enumerate}
Es folgt $V = \underbrace{\widetilde{\eig(\lambda)}}_{V_1} \oplus V_2, \dim(V_2) < n$ und \\
$\alpha = \alpha_1 \oplus \alpha_2, \alpha_i := \alpha|_{V_i}, i\in\{1, 2\}$.
Es folgt $\chi_\alpha = \chi_{\alpha_1} \cdot \chi_{\alpha_2}$, also zerfällt $\chi_{\alpha_2}$
in Linearfaktoren. Daher können wir die Induktionsvorraussetzung anwenden, was das gewünschte Resultat lierfert.
\end{itemize}
\end{proof}
\end{satz}
\begin{satz}
Sei $V \K$-Vektorraum, $\dim(V) < \infty$ und $\alpha \in \homk$ sodass $\chi_A$ in
Linearfaktoren zerfällt. Dann besitzt $\alpha$ Jordan-Normalform.
\begin{proof}
Zerlege nach Satz \ref{theo:2.3.7}
$V = \widetilde{\eig_\alpha(\lambda_1)} \oplus \cdots \oplus \widetilde{\eig_\alpha(\lambda_r)}$
und \\
$\alpha = \alpha \oplus \cdots \oplus \alpha_r$.
Da $\widetilde{\eig_\alpha(\lambda_i)} = \ker(\alpha - \lambda_i \id)^{k_i}$ ist
$\alpha_i - \lambda_i \id := \alpha|_{\widetilde{\eig_\alpha(\lambda_i)}} - \lambda
\id|_{\widetilde{\eig_\alpha(\lambda_i)}} $ nilpotent. Nach Lemma \ref{theo:2.3.3} gibt es eine Basis
$B_i$ von $\widetilde{\eig_\alpha(\lambda_i)}$ sodass ${}_{B_i} M(\alpha_i)_{B_i}$ Jordan-Normalform
hat. Es folgt mit $B= (B_1, \dots, B_r)$ dass ${}_B M(\alpha)_B = \begin{pmatrix}
{}_{B_1} M(\alpha_1)_{B_1} & & \\
& \ddots & \\
& & {}_{B_r}M(\alpha_r)_{B_r}
\end{pmatrix}$ Jordanmatrix ist.
\end{proof}
\end{satz}
\end{document}